Ускорение что такое формула: Ускорение | Физика

Содержание

Формула ускорения – Помощник для школьников Спринт-Олимпик.ру

Суммарный результат всех сил, действующих на объект, как гласит второй закон Ньютона, называется ускорением. Формула его будет изменяться в зависимости от скорости и направления движения предмета. В повседневной жизни этот термин употребляется довольно редко, услышать его можно чаще всего в рекламе автомобилей. Но в физике его значение гораздо шире, чем просто время, за которое машина разгоняется.

Содержание

  • Определение и свойства
  • Другие формы
  • Особые случаи
  • Единица измерения
  • Действие на людей

Определение и свойства

Любое изменение скорости тела приводит к ускорению (ᾱ) как в сторону увеличения, что обычно подразумевается, так и снижения, то есть замедления. Также этот термин может означать смену направления (центростремительность). Это связано с прямой зависимостью сил, которые действуют на объект, от изменения скорости (v), являющейся величиной векторной и имеющей направление. Так ускоряться будут:

  • падающее яблоко;
  • автомобиль, останавливающийся на светофоре;
  • вращающаяся планета и т. п.

Например, транспортное средство начинает движение с места и продолжает ехать, увеличивая v, — это ᾱ линейное (или тангенциальное). Пассажиры внутри машины будут ощущать его как силу, которая прижимает их к спинкам сидений. Если автомобиль поворачивает, то есть меняет направление, то это уже ᾱ радиальное. Люди в салоне будут наклоняться в сторону, противоположную движению.

Когда водитель решит остановиться, это тоже будет ускорением, но только в противоположном направлении v движения авто. В космосе такое ᾱ называют ретроградным горением или замедлением. Пассажиры будут чувствовать, будто что-то их толкает вперёд. Принято различать два вида ᾱ:

  • Среднее. Определяется как изменение скорости (∆v) за какой-либо промежуток времени (∆t). Математическое уравнение выглядит следующим образом: ᾱ = ∆v / ∆t.
  • Мгновенное. Это предел предыдущего ускорения за интервал t, называемый бесконечно малым. Формула будет такая: ᾱ = lim ∆t → 0 * ∆v / ∆t = dv / dt.
  • Например, мотоцикл набирает скорость 50 м/с за 10 с, его среднее ᾱ = 50 / 10 = 5 м/с².

    Другие формы

    Можно взять материальный предмет, например, спутник, который вращается вокруг Земли. Он двигается по окружности и ускоряется, причина этого — изменение направления траектории движения. При этом его скоростной режим может не изменяться. В этом случае речь идёт о центростремительном (направленном к центру) ᾱ.

    Ускорение тела относительно состояния свободного падения (ᾱ правильное) измеряется акселерометром. В механике для предмета с постоянной массой (m) ᾱ центра m тела пропорционально действующему на него вектору силы (суммы всех сил). Здесь действует второй закон Ньютона: F = m * ᾱ → ᾱ = F / m.

    Скорость частицы, которая движется по криволинейной траектории, можно записать как функцию времени v(t) = v(t) * v(t) / v(t) = v(t) * ut(t), где единичный вектор касательной (ut) к траектории равен v(t) / v(t) и указывает направление движения в конкретный момент времени. Это и есть формула центростремительного ускорения, которое создаётся при круговом движении. Можно использовать цепное правило дифференцирования, чтобы записать формулу для произведения двух функций, если принять во внимание, что ᾱ частицы происходит по некой кривой проекции. Последовательность действий уравнения следующая:

  • ᾱ = dv / dt;
  • = dv / dt + v(t) * dut / dt;
  • = dv / dt * ut + v² / r * un.
  • В уравнении un — единичный вектор нормали, r — мгновенный радиус кривизны, который основывается на колеблющемся круге в момент времени t. Все эти компоненты являются тангенциальным, радиальным или нормальным ускорением, формула которого может быть представлена в виде функции.

    Особые случаи

    Если при движении v изменяется на равную величину, то есть объект равноускоренный в каждый одинаковый период времени, то это можно охарактеризовать как равномерное или постоянное ускорение. Пример этого в физике — формула ускорения свободного падения тела, вид которой при отсутствии сопротивления будет зависеть от гравитационного поля и силы стандартной гравитации (g).

    Чтобы составить уравнение, придётся проделать небольшой путь от самых основ. Второй закон Ньютона гласит, что Fg = mg. В кинематике есть формулы, которые связывают смещение (sₒ), начальную (vₒ) и зависящую от времени v(t) скорость и ускорение с прошедшим временем (t):

    • s(t) = sₒ + vₒt + 1/2ᾱt² = sₒ + (vₒ + v(t)/2 * t;
    • v(t) = vₒ² + ᾱt;
    • v²(t) = vₒ² + 2ᾱ * [s(t) — sₒ].

    Наглядно расчёт разности можно увидеть, если начертить график.

    Частица будет испытывать ускорение, которое возникает в результате изменения направления вектора скорости, тогда как её величина остаётся постоянной при равномерном круговом движении. Производная от расположения точки на кривой по времени, то есть её v, оказывается всегда точно касательной к линии, соответствующей ортогональному радиусу в этой точке.

    Это ускорение постоянно меняет направление скорости, которая будет касаться соседней точки, тем самым заставляя вектор скорости совершать вращательные движения по кругу. Формула будет выглядеть следующим образом: ᾱс = v² / r. Надо помнить, что v здесь — произведение угловой скорости ω на r.

    Единица измерения

    Ускорение рассчитывается путём деления метров в секунду (м/с) на секунды (с). Деление расстояния по времени вдвое равно делению расстояния на квадрат времени. Таким образом, единицей ускорения СИ является метр в секунду в квадрате (м/с²). Чтобы было весело изучать физику, можно рассмотреть несколько интересных примеров в таблице.

    ᾱ ( м/с²)Событие
    0,5гидравлический лифт
    0,63ускорение свободного падения (УСП) на Плутоне
    1лифт на кабеле
    1,6ускорение свободного падения на Луне
    8,8Международная космическая станция
    10—40механический прямолинейный старт пилотируемой ракеты
    20космический челнок
    9,8УСП на Земле
    20—50американские горки
    80предел устойчивой человеческой терпимости
    0—150тренировочная центрифуга
    600автоматические подушки безопасности
    1 млнпуля в стволе пистолета
    24,8УСП на Юпитере

    Другая часто используемая единица — ускорение силы тяжести g. Поскольку все знакомы с влиянием гравитации на физические объекты, это делает их удобным стандартом для сравнения ускорений. Все чувствуют себя нормально при 1 g, вдвое тяжелее при 2 g и невесомо при 0 g. Эта единица измерения имеет значение 9,80665 м/с², но для повседневного использования достаточно 9,8 м/с², а 10 м/с² удобно для быстрых подсчётов.

    Действие на людей

    Хотя термин «сила g» часто используется, g — мера ускорения, а не силы. Особую обеспокоенность у людей вызывают физиологические эффекты этого явления. Чтобы понять смысл, лучше обратиться к примерам:

  • Все знают аттракцион «Американские горки». Скорость там очень важна. Если бы она была единственной целью проектировщиков острых ощущений, то автострада оказалась бы довольно захватывающей. Однако всё очень скромно, многие горки редко превышают скоростной режим, равный 30 м/с (примерно 97 км/ч). Вопреки распространённому мнению, именно ускорение делает поездку интересной. Тщательно разработанные горки позволят пассажирам на короткое время максимально ускориться (как равноускориться, так и равнозамедлиться) от 3 до 4 g — это то, что даёт поездке ощущение опасности.
  • Несмотря на огромную мощность своих двигателей, разгон космического модуля удерживается ниже 3 g, поскольку всё, что больше, создаёт ненужную нагрузку на космонавтов и сам корабль. Оказавшись на орбите, вся система вступает в длительный период свободного падения, что даёт ощущение невесомости. Такое чувство также может быть смоделировано внутри специально пилотируемого самолёта или башни свободного падения.
  • Пилоты истребителей могут испытывать ускорение до 8 g в течение коротких периодов во время тактических манёвров. Если воздействие продлится более нескольких секунд, то будет достаточно 4—6 g, чтобы вызвать потерю сознания. Для предотвращения таких ситуаций лётчики-истребители носят специальную одежду, которая сжимает ноги и живот, заставляя кровь приливать к голове.
  • Пилоты и космонавты могут также тренироваться на специальных центрифугах, способных выдавать до 15 g. Воздействие таких интенсивных ускорений является кратким из-за соображений безопасности.
  • Ускорение травмоопасно, поэтому наиболее распространённым датчиком манекена для краш-теста является акселерометр. Чрезвычайное ускорение может привести к смерти.
  • По оценкам экспертов, ускорение во время аварии, в которой погибла принцесса Диана, составляло порядка 70—100 g.

    Этого было достаточно, чтобы оторвать лёгочную артерию от её сердца и спровоцировать травму, которую практически невозможно пережить. Если бы Диана была пристёгнута ремнём безопасности, ускорение составило бы примерно 30 или 35 g. Это грозило несколькими переломами, но все остались бы живы.

    Предыдущая

    ФизикаЭлектрический ток в электролитах – механизм возникновения, законы и применение

    Следующая

    ФизикаНаправление электрического тока – условия и причины возникновения

    Как рассчитать ускорение с помощью силы и массы: исчерпывающие подходы и факты

    Как рассчитать ускорение с помощью силы и массы – это известный вопрос, на который нужно ответить. Мы знаем ускорение, которое действует на тело, где сила и масса существенно влияют на него.

    Ускорение измеряет, насколько скорость изменяется в данный момент времени. Скорость – это фактическая скорость, с которой объект изменяется в соответствии с системой отсчета. Скорость зависит от того, какая сила перемещает объект вперед или назад.

    Ускорение, действующее на тело, в основном зависит от силы и массы, и как рассчитать ускорение с помощью силы и массы – это то, что мы обсудим далее. Когда неуравновешенная сила не равна нулю, действие на тело будет иметь большее ускорение, когда чистая сила не равна нулю.

    Когда равнодействующая сила действующее на тело больше, ускорение, действующее на тело, также будет больше. Еще один момент, который следует помнить, это то, что масса также влияет на ускорение. Чем меньше масса, тем больше ускорение.

    Как мы все знаем, законы Ньютона были применены к нескольким открытиям, а для ускорения мы используем Второй закон Ньютона. Это удобно, когда мы используем его для расчета ускорения любого движущегося тела.

    Согласно второму закону Ньютона сила, действующая на тело, пропорциональна массе, постоянной во всем и изменяющейся скорости. Формула имеет вид F = ма. Мы получаем ускорение как а = Ф / м.

    В этом контексте мы рассматриваем заставить быть сетью сила. Потому что на тело действует несколько сил, поддерживающих его движение, например, нормальная сила, сила трения и так далее. Следовательно, мы считаем, что полная сила, действующая на тело, является равнодействующей силой.

    Как найти ускорение по чистой силе и массе

    Во-первых, нам нужно понять, как сила и масса влияют на ускорение движущегося тела. Масса – это вес движущегося тела, а сила – это не что иное, как чистая сила, действующая на тело, которая запускает его движение.

    Все это возможно только благодаря одной формуле, которая подчиняется Второму закону Ньютона и выводится из него. Формула выглядит так: а = Ф / м.

    Здесь сила имеет разные случаи; сила может быть нормальной, силой трения, силой натяжения, силой тяжести, Равнодействующая сила, и чистая сила. Здесь, в этом случае, мы рассматриваем силу как результирующую силу. И эта конкретная сила случается с неуравновешенной силой.

    При воздействии на тело эта неуравновешенная сила заставляет тело менять свое положение покоя, из которого оно начинает двигаться. Тогда у тела не будет другого выбора, кроме как совершить движение.

    Простые примеры, иллюстрирующие, как найти ускорение с помощью силы и массы

    Когда прикладываемая масса меньше, ускорение больше и наоборот. Возьмем небольшой пример мяча массой 10 кг, катящегося под гору с силой 40 Н. Поэтому мы также должны учитывать направление, в котором движется мяч. Пусть она будет перпендикулярна зоне, по которой движется.

    Согласно уравнению Второго закона Ньютона, f = mxa

    Итак, a = F / m

    а = 40/10

    a = 4 мс-2

    Давайте также проиллюстрируем другой пример для лучшего понимания. В деревне был холм, на котором мало кто пользовался, потому что на нем было слишком много камней и мелких камней, которые могли скатиться и поранить пешеходов.

    “горные породы” by стебул под лицензией CC BY 2.0

    Однажды из-за проливных дождей скала и камни больше не оставались неподвижными и начали быстро катиться с холма. Теперь давайте посчитаем ускорение большого камня массой 500 кг, который скатывается с силой 1500 Н.

    По формуле a = F / m

    а = 15000/500

    a = 30 мс-2

    Помимо ускорения с силой и массой, есть еще один решающий фактор: сила тяжести. Кроме того, в этом разделе мы подробно рассмотрим реальный пример того, как вычислить ускорение с помощью силы и массы.

    Группа мальчиков приехала в деревню на летние каникулы. Один из мальчишек из деревни очаровал своего друга рекой, имеющей прыгающий конец. Следовательно, все мальчики захотели попробовать. Один за другим прыгали в реку с прыжковой точки.

    Теперь посчитайте ускорение мальчика весом 30 кг, который прыгает в реку с силой 120 Н. Мы знаем формулу, а = Ф / м. Следовательно, a = 120/30, a = 3 мс-2.

    Ускорение с силой и массой в повседневной жизни

    Всегда есть любопытство по поводу того, как все работает и воплощается в реальность. Точно так же мы обычно видим движение транспортного средства, но задумывались ли мы, как физики, о физике, лежащей в основе этой причины?

    Каждый день то, что мы видим, воспринимаем физику, но не осознаем ее. Как было сказано ранее, мы видим машину, движущуюся или даже едущую в ней; мы никогда не замечаем науки, стоящей за этим процессом. Поэтому главное, что нужно делать, – это замечать и признавать их в нашей повседневной жизни.

    Ускорение движения тела и выводы – это то, что мы рассмотрим в этом разделе. Итак, с этого момента, где бы и когда бы мы ни увидели движущийся автомобиль или автобус, мы немедленно должны знать, что ускорение способствует такому процессу.

    Во-первых, нам нужно увидеть, какое отношение масса имеет к ускорению. Масса – это вес того конкретного тела, который связан с ускорением. Когда масса мала, ускорение больше. Следовательно, вот как рассчитать ускорение с силой и массой.

    Это просто здравый смысл; когда предмет тяжелый, его сложно переместить с одного места на другое. Когда масса легкая, ее можно быстро мобилизовать. Следовательно, масса имеет прямую связь с ускорением.

    «Автомобиль N Motion» by TheBusyBrain под лицензией CC BY 2.0

    Задачи о том, как рассчитать ускорение с помощью силы и массы

    1 задачи:

    Кафе находится на углу города. Каждое утро несколько велосипедистов проезжают по городу и пересекают кафе. Скамейки кафе вынесены на улицу, поэтому вид улучшается, так как это пляжное кафе.

    Однажды один из велосипедистов ехал слишком быстро, двинулся и слегка задел гигантскую статую, расположенную рядом. Эта статуя из-за своего тяжелого веса медленно упала на землю. Теперь посчитайте, с каким ускорением статуя весом 800 кг обладает силой тяжести 1100 Н.

    Решение:

    а = Ф / м

    а = 1100/800

    a = 1.375 мс-2

    2 задачи:

    В данный момент объект неподвижен. Когда чистая сила 175 Н действует на этот объект весом 50 кг, какое ускорение он будет использовать, чтобы выйти из своего исходного положения?

    Решение:

    а = Ф / м

    а = 175/50

    a = 3.5 мс-2

    Часто задаваемый вопрос

    Какие несколько примеров ускорения в повседневной деятельности?

    Ускорение – это скорость, с которой объект движется в данный момент времени. Это зависит от изменения скорости с заданным временем.

    • Когда объект движется на юг на 10N м / с с постоянной скоростью, он остается в том же темпе до тех пор, пока не будет приложена сила. Таким образом, при приложении силы он будет двигаться со скоростью 2 мс-15. Теперь объект называется ускоряющимся.
    • Девушка идет на север со скоростью 10 мс-2. Говорят, что у девушки постоянная скорость, а значит, и ускорение равно нулю.
    • Когда мяч катится с холма, он ускоряется при приложении силы. Если мяч весит меньше, ускорение больше, а если мяч весит больше, ускорение со временем становится меньше.
    • Когда мальчик прыгает со скалы с приложенной к нему силой, он будет ускоряться еще больше, а затем, наконец, остановится под действием силы тяжести. Но величина необходимого ускорения будет зависеть от прилагаемой силы: если сила увеличивается, то ускорение увеличивается; если сила уменьшается, ускорение уменьшается.  
    • Мальчик движется на север с ускорением 9 мс.-2. Постепенно на мальчика действует другая сила, и теперь размер меняется на 15 мс.-2. Теперь говорят, что мальчика ускоряют за счет действующей на него чистой силы.

    Что такое ускорение? Виды ускорения. Формулы. Пример решения задачи

    11.12.2018 15:00

    Раздел физики, ответственный за изучение особенностей движения в пространстве тел, называется кинематикой. В данной статье рассмотрим, какие физические величины в кинематике используются для описания перемещений объектов, а также раскроем, что такое ускорение.

    Физические величины в кинематике

    Когда тело движется в пространстве, то нам важно знать, какое расстояние оно проходит за указанный промежуток времени и вдоль какой траектории движется.

    Для описания пройденных расстояний в физике используют понятие пути – L. В случае движения по окружности вместо пути пользуются понятием угла поворота – θ. Величину L в СИ измеряют в метрах (м), а величину θ – в радианах (рад.).

    Помимо пути важно знать также скорость движения тел. Под ней понимают быстроту прохождения расстояний. Математическое выражение для линейной скорости принимает вид:

    v¯ = d L / d t

    Для описания движения по окружности применяют угловую скорость ω, которая рассчитывается так:

    ω¯ = d θ / d t

    Третьей важной величиной кинематики является ускорение.

    Что такое ускорение? Это величина в физике, которая показывает, как быстро меняется скорость во времени. Математически это можно записать так:

    a¯ = d v¯ / d t

    Если подставить в эту формулу ускорения выражение для скорости, получим:

    a¯ = d2 L / d t2

    Ускорение – это первая производная скорости по времени или вторая производная по времени пройденного пути.

    Тангенциальное и нормальное ускорение

    Выше было дано определение, что такое ускорение. Оно называется полным. В общем случае направление полного ускорения не совпадает с направлением вектора скорости. Последний является касательной к траектории движения в любой ее точке.

    Поскольку скорость – это величина векторная, то ее изменение предполагает возможность менять модуль и направление. В первом случае говорят о наличии у тела тангенциального ускорения, во втором – нормального.

    Формула тангенциального ускорения at не отличается от таковой для ускорения полного a. Формула имеет вид:

    at = d v / d t

    То есть тангенциальное, или касательное, как его еще называют, ускорение является производной от модуля скорости по времени. Вектор at¯ совпадает с вектором v¯ при ускоренном движении и противоположен ему при замедленном движении.

    Нормальное ускорение – это физическая величина, которая приводит к искривлению прямолинейной траектории перемещения тел. Направлено оно вдоль радиуса кривизны траектории, то есть нормально по отношению к ней. Формула для его определения имеет вид:

    ac = v2 / r

    Нормальное ускорение ac зависит от модуля скорости v и радиуса кривизны траектории r. Очевидно, что в случае движения по прямой радиус r можно считать равным бесконечности. Последнее означает, что нормальное ускорение равно нулю для прямолинейного движения.

    Для движения по окружности вектор ac¯ направлен к ее центру вдоль радиуса. По этой причине величину ac также называют центростремительным ускорением.

    Полное ускорение

    Вектор полного ускорения – это всегда сумма тангенциальной и нормальной компонент. Поскольку они перпендикулярны друг другу, то для вычисления модуля полного ускорения можно воспользоваться теоремой Пифагора. Искомая формула ускорения полного примет вид:

    a = √(at2 + ac2)

    Чтобы определить, куда направлен вектор a¯, достаточно вычислить угол между ним и какой-либо компонентой. Например, угол φ между векторами a¯ и at¯ равен:

    φ = arctg(ac / at)

    Напомним, что центростремительное ускорение отлично от нуля только тогда, когда кривизна траектории движения отлична от бесконечности. В случае же прямолинейного движения полное ускорение по величине и направлению равно тангенциальной компоненте.

    Угловое ускорение

    Рассматривая, что такое ускорение, следует остановиться на соответствующей угловой характеристике.

    Выше было введено понятие угловой скорости, которая измеряется в радианах в секунду (рад/с). Если найти производную этой скорости по времени, то мы получим величину углового ускорения:

    α¯ = ω¯ / d t

    Несложно показать, что угловая величина связана с тангенциальной компонентой полного ускорения следующим соотношением:

    at = α × r

    При постоянном угловом ускорении касательная компонента at будет больше для точек, которые находятся дальше от оси вращения.

    К нормальной компоненте угловое ускорение не имеет никакого отношения.

    Решение задачи на определение ускорения

    Предположим, что, двигаясь с ускорением постоянным вдоль прямой линии, тело прошло расстояние 100 метров. Известно, что начальная скорость тела была равна 1 м/с. Отмеченное расстояние тело преодолело за 5,5 секунды. С каким ускорением происходило движение?

    Согласно условию задачи, речь идет о равноускоренном движении вдоль прямой траектории. Пройденный путь в этом случае может быть вычислен по формуле:

    L = v0 × t + a × t2 / 2

    Выражаем из равенства величину a, имеем:

    a = 2 × (L – v0 × t) / t2

    Все величины в правой части равенства известны из условия. Подставляем их и записываем ответ: a = 6,25 м/с2. То есть в течение каждой из 5,5 секунд скорость тела возрастает на 6,25 м/с. Найденное значение полного ускорения совпадает с тангенциальной компонентой.

    Формула пересчета ускорения из движущейся s – со в «неподвижную» s

    СЕМИНАР 1 Кинематика точки Вектор скорости, модуль вектора скорости, вектор ускорения, модуль вектора ускорения.

    Тангенциальное ускорение.

    – тангенциальное ускорение – производная от модуля скорости по времени. Для нахождения тангенциального ускорения сначала находим модуль скорости как функцию времени и затем дифференцируем эту функцию по времени.

    Нормальное ускорение.

    Вектору скорости присущи два атрибута: модуль и направление в пространстве. Производная вектора скорости по времени, может быть представлена в виде суммы двух слагаемых. Одно из этих слагаемых – это тангенциальное (касательное) ускорение.

    Другое слагаемое характеризует быстроту изменения направления скорости – это нормальное ускорение. Таким образом, имеем .

    В соответствии с теоремой Пифагора, получаем полезную формулу .

    Радиус кривизны траектории.

    Можно показать, что нормальное ускорение, характеризующее быстроту изменения направления скорости, связано с величиной скорости формулой . Здесь ρ – радиус кривизны траектории. Отсюда получаем . Именно такой формулой будем пользоваться для нахождения радиуса кривизны траектории в этом разделе.

    Вращательное движение твердого тела вокруг постоянной оси

    Угловая скорость, угловое ускорение.

    При описании вращательного движения твердого тела, наряду с векторами перемещения любых точек твердого тела, вводят единый для всех точек вектор элементарного угла поворота . Кроме линейных скоростей точек твердого тела, вводят единую для всех точек угловую скорость . Угловое ускорение . Формула, связывающая величину угловой скорости и частоты вращения.

    Связь угловых характеристик движения с линейными.

    ,; , ; , . Здесь – радиус – вектор, рассматриваемой точки твердого тела, начинающийся в любой точке оси вращения; R – расстояние от рассматриваемой точки твердого тела до оси вращения.

    Кинематика относительного движения (Галилей, Кориолис)

    , – скорость и ускорение материальной точки относительно S – СО; , – скорость и ускорение материальной точки относительно S – СО; – радиус-вектор материальной точки относительно S – СО; , – скорость и ускорение S

    – СО относительно S – СО в поступательном движении; , – угловая скорость и угловое ускорение S – СО относительно S – СО во вращательном движении. Тогда формула пересчета скорости из движущейся S – СО в «неподвижную» S – СО имеет вид: , то есть, скорость материальной точки относительно “неподвижной” S – СО складывается из скорости материальной точки относительно движущейся S – СО и скорости точки S – СО, через которую проходит (в этот момент) материальная точка, относительно S – СО.

    тоже утверждает, что ускорение материальной точки относительно “неподвижной” S – СО складывается из ускорения материальной точки относительно движущейся S – СО и ускорения точки S – СО, через которую проходит (в этот момент) материальная точка, относительно S – СО. Кориолисово ускорение . Оно связано, во-первых, с тем, что вектор поворачивается вместе с S – СО и, во-вторых, с тем, что из-за перемещения материальной точки относительно S – СО, изменяется радиус-вектор , а значит и скорость .

    СЕМИНАР 2 ДИНАМИКА МАТЕРИАЛЬНОЙ ТОЧКИ И ПОСТУПАТЕЛЬНО ДВИЖУЩЕГОСЯ ТВЕРДОГО ТЕЛА

    Инерциальные системы отсчета

    Важная роль выбора системы отсчета впервые продемонстрирована Коперником (около 1500г.). В системе отсчета введенной Коперником, связанной с Солнцем и звездами, настолько упростился характер движения планет, что трудолюбивый Кеплер (в 1609-1619гг. ) сумел сформулировать три знаменитых закона, описывающих движение планет. Следуя Копернику, Ньютон навсегда в качестве тел отсчета выбрал Солнце и звезды. Опираясь на законы Кеплера, Ньютон установил закон всемирного тяготения, а затем и три закона движения (около 1666г.). Все это было сделано применительно к коперниковой (гелиоцентрической), инерциальной системе отсчета.

    Первый закон Ньютона содержит не только закон инерции Галилея, но и определение инерциальной системы отсчета:

    Существуют такие системы отсчета, назовем их инерциальными (ИСО), в которых тело, изолированное от других тел, сохраняет свою скорость постоянной.

    Нахождение силы из закона движения.

    Импульсом материальной точки называется величина, равная произведению массы точки на ее скорость . По определению, сила – это величина, показывающая, как быстро изменяется импульс материальной точки со временем, то есть , причем последние два равенства справедливы, если масса тела постоянна.

    Интегрирование уравнения движения. Сила линейно зависит от времени.

    – уравнение движения материальной точки в векторной форме. В проекции на оси прямоугольной системы координат уравнения движения принимают вид ; ;

    Интегрируем соответствующее дифференциальное уравнение методом разделения переменных.

    Интегрирование уравнения движения. Сила зависит от координаты.

    В уравнении движения делаем замену. Тогда уравнение принимает вид , то есть переменные разделились и можно выполнить интегрирование.

    Неинерциальные системы отсчета

    Система отсчета, относительно которой материальная точка движется с ускорением, при условии, что на эту точку не действуют другие тела, называется неинерциальной (НСО).

    Можно сказать иначе. Система отсчета, которая движется поступательно с ускорением и/или вращается относительно инерциальной системы отсчета (ИСО), называется неинерциальной (НСО).

    Введем следующие обозначения:

    , – скорость и ускорение материальной точки относительно неинерциальной S – СО;

    – радиус-вектор материальной точки относительно неинерциальной S – СО;

    – ускорение неинерциальной S – СО относительно инерциальной S – СО в поступательном движении;

    , – угловая скорость и угловое ускорение неинерциальной S – СО относительно инерциальной S – СО во вращательном движении.

    В этих обозначениях уравнение движения материальной точки в неинерциальной системе отсчета имеет вид: .

    В правой части уравнения:

    – сумма всех сил, действующих на материальную точку со стороны других тел, то есть тех сил, которые определены в рамках системы законов Ньютона;

    – сила инерции, действующая в НСО, движущейся поступательно с ускорением ;

    – сила инерции, действующая в НСО, вращающейся с угловым ускорением;

    – центробежная сила инерции, действующая в НСО, вращающейся с угловой скоростью ;

    – сила инерции Кориолиса, действующая в НСО, вращающейся с угловой скоростью , если материальная точка движется относительно НСО со скоростью и при условии, что векторы и составляют угол, не равный 0

    0 или 1800.

    СЕМИНАР 3 ИМПУЛЬС, МЕХАНИЧЕСКАЯ ЭНЕРГИЯ И МОМЕНТ ИМПУЛЬСА (ЗАКОНЫ ИЗМЕНЕНИЯ И СОХРАНЕНИЯ)

    Закон изменения импульса для одной материальной точки.

    Второй закон Ньютона для материальной точки, когда на нее действует постоянная сила, может быть переписан в виде закона изменения импульса – приращение импульса материальной точки равно импульсу силы (произведению силы на время, за которое импульс точки изменился на), действующей на материальную точку.

    Система материальных точек.

    Импульс системы материальных точек – это сумма (конечно векторная) импульсов материальных точек: . Производная импульса системы материальных точек по времени равна сумме всех сил, действующих на систему, и, с учетом третьего закона Ньютона, равна сумме внешних сил, действующих на систему материальных точек: .

    Сохранение импульса системы взаимодействующих тел.

    Из закона изменения импульса следует, что если , то. Для проекций на выделенное направление X можно утверждать, что из следует , если .

    Уравнение движения тела с изменяющееся массой – уравнение Мещерского Здесь m – масса, – ускорение тела в рассматриваемый момент времени, – сумма всех внешних сил, – реактивная сила.

    Центр масс. Система отсчета центра масс.

    Центром масс системы материальных точек называется точка пространства, радиус-вектор которой находится по формуле . Соответственно скорость центра масс равна . Системой отсчета центра масс (Ц-системой) называется такая система отсчета, относительно которой покоится центр масс рассматриваемой системы частиц, и, которая движется поступательно относительно инерциальной системы отсчета.

    Работа постоянной силы.

    – работа постоянной силы, приложенной к телу, определяется как скалярное произведение вектора силы на вектор перемещения тела.

    Работа переменной силы.

    Разделяем конечное перемещение на такие элементарные перемещения, чтобы на любом из них можно было считать силу постоянной по величине и по направлению. Тогда можно ввести понятие элементарной работы . Затем учитываем замечательное свойство работы – аддитивность (свойство складываться): .

    Мощность силы .

    Теорема о приращении кинетической энергии. – приращение кинетической энергии материальной точки или поступательно движущегося твердого тела равно работе всех сил, приложенных к материальной точке или к телу.

    Потенциальная энергия взаимодействия системы материальных точек.

    Для того, чтобы работа силы, приложенной к телу, при переносе тела из позиции 1 в позицию 2 не зависела от формы траектории, необходимо, чтобы сумма была полным дифференциалом. В свою очередь, для того, чтобы указанная сумма была полным дифференциалом, должны выполняться равенства ; ; . Только при выполнении этих условий можно сопоставить точкам пространства некоторую функцию координат и назвать ее потенциальной энергией, а силу потенциальной или консервативной. Определение формулируется не для потенциальной энергии, а для ее приращения , или ее убыли .

    Таким образом, потенциальная энергия неопределенна с точностью до постоянной – уровня отсчета потенциальной энергии. Определение приращения потенциальной энергии в дифференциальной форме имеет вид . Отсюда .

    Физика от чего зависит ускорение

    Содержание

    • 1 Основные формулы для равноускоренного движения
    • 2 Сила тяжести и ускорение свободного падения
    • 3 Закон всемирного тяготения
    • 4 Как определяют ускорение силы тяжести
    • 5 Что мы узнали?
    • 6 История открытия
    • 7 Физическая сущность
    • 8 Формулы для расчёта
    • 9 Ускорение на других планетах
    • 10 Воздействие перегрузок на человека

    Основные формулы для равноускоренного движения

    Равноускоренное движение — самый простой вид неравномерного движения. Равноускоренным называется движение с ускорением, постоянным по модулю и направлению:

    Δv — изменение скорости (“дельта v “), м/с;

    Δt — промежуток времени, (“дельта t “)за которое произошло изменение скорости, с.

    Из формулы (1) следует, что размерность ускорения будет выражаться в метрах на секунду в квадрате:

    Второй закон Ньютона гласит:

    F — сила, действующая на тело, Н;

    m — масса тела, кг;

    a — ускорение, м/с 2 .

    Сила тяжести и ускорение свободного падения

    При свободном падении на Землю все тела, независимо от их массы, движутся одинаково. Свободное движение является равноускоренным движением. Ускорение, с которым падают на Землю тела в пустоте, называется ускорением свободного падения (или ускорением силы тяжести). Условие пустоты или, что тоже самое, вакуума, требуется для исключения влияния сопротивления атмосферного воздуха. Сила притяжения Fт со стороны Земли на тело массой m, называется силой тяжести:

    Определением ускорения силы тяжести впервые систематически занимался Галилео Галилей — итальянский математик, физик, астроном. 2>$ .

    И хотя это открытие датировано 1589г., современное, общепринятое среднее значение g практически не отличается от этого значения. Когда от расчетов не требуется высокой точности, то принимают, что модуль g равен 10 м/с 2 .

    Последовавшие за Галилеем более точные измерения показали, что значение g не является абсолютной константой, а зависит от местоположения измерений в разных точках Земли. Ответ на этот вопрос нашел английский ученый Исаак Ньютон.

    Закон всемирного тяготения

    В 1682 г. Ньютон открыл закон всемирного тяготения, из которого следует:

    • все тела притягиваются друг к другу;
    • сила тяготения прямо пропорциональна произведению масс тел и обратно пропорциональна квадрату расстояния между ними;
    • векторы сил тяготения направлены вдоль прямой, соединяющей тела.

    Этот закон универсален, и для случая пары тел, одно из которых является произвольным телом массой m, а второй — Земля, в виде формулы выглядит так:

    Mз — масса Земли, кг;

    Rз — радиус Земли, м;

    h — высота, на которой находится тело, относительно поверхности Земли, м;

    G — гравитационная постоянная, равная 6,6720 * 10 -11 Н*м 2 * кг -2 .

    Из формул (4) и (5) следует, что:

    Из (6) следует, что ускорение силы тяжести будет зависеть от высоты h и величины радиуса Земли, который для обычных расчетов принимается равным примерно 6400 км. Но поскольку форма Земли не является идеальным шаром, а сплюснута к полюсам, то точные значения g будут отличаться от среднего значения в 9,81 м/с 2 :

    • максимальное значение gмакс = 9,83 м/с 2 — на полюсах Земли, где Rз меньше;
    • минимальное значение gмин = 9,79 м/с 2 — на экваторе Земли, где Rз больше.

    Рис. 2. Зависимость ускорения свободного падения на полюсах, экваторе и от вращения Земли.

    Из формулы (6) также следует, что ускорение силы тяжести на других планетах, имеющих массу, отличающуюся от массы Земли, будет для космонавтов значительно отличаться от привычных земных условий. Так, например:

    • На Марсе — gМарса = 3,86 м/с 2 ;
    • На Меркурии — gМеркурия = 3,7 м/с 2 ;
    • На Луне — gЛуны = 1,62 м/с 2 ;
    • На Нептуне — gНептуна = 11,0 м/с 2 .

    Как определяют ускорение силы тяжести

    Для точного измерения силы тяжести, а значит, и ускорения, используется прибор, называемый гравиметром. Прибор применяется при поиске полезных ископаемых и для сбора информации археологами, палеонтологами, гидрологами и представителями других профессий, изучающих поверхность Земли.

    Рис. 3. Гравиметры:.

    Следует упомянуть еще два фактора, влияющих на значение ускорения свободного падения:

    • Известно, что Земля вращается вокруг своей оси, имея при этом так называемое центростремительное ускорение, которое влияет на величину ускорения свободного падения;
    • Масса Земли распределена неравномерно, например, в местах расположения больших месторождений металлических руд ускорение силы тяжести будет больше, а там, где есть пустоты (газовые месторождения) ускорение будет несколько меньше.

    Эти факторы дают очень малые отклонения от средних значений g , но зато их регистрация позволяет, например, геологам находить новые месторождения полезных ископаемых.

    Что мы узнали?

    Итак, мы узнали, что такое ускорение силы тяжести. Сила тяжести возникает вследствие действия силы гравитации, подчиняющейся закону Ньютона (формула (5)). На Земле среднее значение ускорения силы тяжести gЗемли равно 9,81 м/с 2 . Для точного определения ускорения силы тяжести требуется использование современных приборов, называемых гравиметрами.

    ФИЗИКА

    Вопрос 1. Кинематика материальной точки (частицы). Скорость и ускорение при одномерном движении точки. Выражение для перемещения ΔX через скорость Vx. Выражение для ΔVx через ускорение .

    Механика – часть науки о природе. Одним из разделов механики является кинематика. Кинематика – это раздел механики, посвященный изучению геометрических свойств движений без учета их масс и действующих на них сил. Основная задача механики – определить положение тела в любой момент времени. Время – непрерывно и неоднородно меняющаяся переменная. Положение тела – положение тела относительно других тел. Тело, размерами которого можно пренебречь в условиях данной задачи, называется материальной точкой. Совокупность тел, выделенная для рассмотрения, называется механической системой. Совокупность неподвижных друг относительно друга тел, по отношению к которым рассматривается движение, и отсчитывающих время часов образуют систему отсчета. Для того чтобы получить возможность описывать движение количественно, приходится связывать с телами, образующими систему отсчета, какую-либо систему координат, например, декартову.

    Абсолютно твердое тело – тело, деформациями которого в условиях данной задачи можно пренебречь.

    Движение твердого тела:

    1) поступательное –это такое движение, при котором любая прямая, связанная с движущимся телом, остается параллельной самой себе.

    2) вращательное – это такое движение, при котором все точки тела движутся по окружностям, центры которых лежат на одной и той же прямой, называемой осью вращения. Ось вращения может находиться вне тела.

    Одномерным называется движение тела, при котором его положение в пространстве может быть полностью охарактеризовано при помощи одной координаты (например, положение поезда можно задать, указав расстояние вдоль железнодорожного полотна до станции отправления). Прямолинейное движение является важнейшим частным случаем одномерного. При движении тела его координата изменяется во времени, на языке математики это означает, что координата является функцией аргумента t.

    Эту функцию можно задать при помощи таблицы, графика, аналитического выражения.

    Для одномерного равноускоренного движения (скорость и ускорение):

    ΔVx= Δt

    Вопрос 2. Средняя скорость и среднее ускорение при прямолинейном движении частицы. Путь в случае изменения направления движения.

    Средней скоростью называют отношение Δ к Δt.

    = Δ / Δt

    Δ — перемещение тела

    сонаправлена с Δ .

    Среднее ускорение — физическая величина, численно равная отношению изменения скорости ко времени, за которое оно произошло:

    Криволинейное равноускоренное (равнопеременное) движение также можно рассматривать как одномерное. В этом случае используется обобщенная координата S, часто называемая путем. Эта координата соответствует длине пройденной траектории (длине дуги кривой). Таким образом, формула приобретает вид:

    Вопрос 3. Равнопеременное прямолинейное движение. Выражения для ΔVx и Δx для равноускоренного движения.

    Равнопеременное движение – это движение, при котором скорость тела (материальной точки) за любые равные промежутки времени изменяется одинаково.

    Ускорение тела при равнопеременном движении остаётся постоянным по модулю и по направлению (a = const).

    Равнопеременное движение может быть равноускоренным или равнозамедленным.

    Равноускоренное движение – это движение тела (материальной точки) с положительным ускорением, то есть при таком движении тело разгоняется с неизменным ускорением. В случае равноускоренного движения модуль скорости тела с течением времени возрастает, направление ускорения совпадает с направлением скорости движения.

    Равнозамедленное движение – это движение тела (материальной точки) с отрицательным ускорением, то есть при таком движении тело равномерно замедляется. При равнозамедленном движении векторы скорости и ускорения противоположны, а модуль скорости с течением времени уменьшается.

    В механике любое прямолинейное движение является ускоренным, поэтому замедленное движение отличается от ускоренного лишь знаком проекции вектора ускорения на выбранную ось системы координат.

    ΔVx= Δt

    Вопрос 4*. Равнопеременное прямолинейное движение ( = const). Выражения для Vx(t) и x(t). Графики этих функций при различных знаках vx(t) и . Графический смысл перемещения Δx и ускорения на графике vx(t).

    Равнопеременное движение – это движение, при котором скорость тела (материальной точки) за любые равные промежутки времени изменяется одинаково.

    Ускорение тела при равнопеременном движении остаётся постоянным по модулю и по направлению (a = const).

    Равнопеременное движение может быть равноускоренным или равнозамедленным.

    Равноускоренное движение – это движение тела (материальной точки) с положительным ускорением, то есть при таком движении тело разгоняется с неизменным ускорением. В случае равноускоренного движения модуль скорости тела с течением времени возрастает, направление ускорения совпадает с направлением скорости движения.

    Равнозамедленное движение – это движение тела (материальной точки) с отрицательным ускорением, то есть при таком движении тело равномерно замедляется. При равнозамедленном движении векторы скорости и ускорения противоположны, а модуль скорости с течением времени уменьшается.

    В механике любое прямолинейное движение является ускоренным, поэтому замедленное движение отличается от ускоренного лишь знаком проекции вектора ускорения на выбранную ось системы координат.

    Формула скорости равнопеременного движения в любой момент времени:

    = + t

    Если тело движется прямолинейно вдоль оси ОХ прямолинейной декартовой системы координат, совпадающей по направлению с траекторией тела, то проекция вектора скорости на эту ось определяется формулой:

    Знак «-» (минус) перед проекцией вектора ускорения относится к равнозамедленному движению. Аналогично записываются уравнения проекций вектора скорости на другие оси координат.

    Так как при равнопеременном движении ускорение является постоянным (a = const), то график ускорения – это прямая, параллельная оси 0t (оси времени).

    Зависимость ускорения тела от времени.

    Зависимость скорости от времени – это линейная функция, графиком которой является прямая линия.

    График зависимости скорости от времени показывает, что

    При этом перемещение численно равно площади фигуры 0abc.

    Площадь трапеции равна произведению полусуммы длин её оснований на высоту. Основания трапеции 0abc численно равны:

    Высота трапеции равна t. Таким образом, площадь трапеции, а значит, и проекция перемещения на ось ОХ равна:

    В случае равнозамедленного движения проекция ускорения отрицательна и в формуле для проекции перемещения перед ускорением ставится знак «–» (минус).

    Общая формула для определения проекции перемещения:

    Дата добавления: 2018-04-15 ; просмотров: 734 ; ЗАКАЗАТЬ РАБОТУ

    История открытия

    Учёные Древней Греции разделяли любое движение на два типа: естественное и принудительное. Перемещение тела под воздействием гравитации считалось естественным, так как не имело видимой причины и происходило само собой.

    Аристотель считал, что скорость падения напрямую зависит от массы. Это ошибочное утверждение родилось в результате примитивных наблюдений. Философ приводил в пример движение к земле яблок и листьев. Очевидно, что последние летели гораздо медленнее. Исследователи тех времён ещё очень мало понимали в физике. Такие понятия, как сопротивление воздуха и ускорение были неизвестны.

    Утверждения Аристотеля считались неоспоримым постулатом вплоть до начала XVII века. Галилео Галлилей решительно отверг древнюю классификацию движения. В результате проведения нескольких опытов с движением тела по наклонной плоскости, учёный ввёл понятие ускорения.

    Определение ускорения свободного падения в физике

    Основное внимание Галлилей уделял изучению процесса свободного падения. Самым знаменитым стал эксперимент, проведённый на Пизанской башне.

    С сооружения высотой 60-м были одновременно сброшены два предмета:

    • маленький металлический шарик весом в пол фунта;
    • большая круглая бомба, весившая 100 фунтов.

    Результат был просто ошеломляющим. Оба тела достигли земли практически одновременно, а небольшая разница была объяснена силой сопротивления воздушной среды. Надо заметить, что наука тех лет существенно отличалась от сегодняшней. Считалось, что воздух не мешает падению, а, напротив, увеличивает его скорость.

    Ещё одним заблуждением того времени было утверждение о том, что любое движение со временем прекращается, даже если на его пути нет преград. Галлилей опроверг и этот ошибочный закон физики, введя определение инерции.

    В XVI веке ещё не существовало точных хронометров. Из-за этого ускорение падения тел с Пизанской башни было рассчитано довольно грубо. Для более точного измерения учёный изучал равноускоренное движение шарика по наклонной плоскости. А более или менее правильное значение ускорения сумел вычислить Гюйгенс в 1660 г.

    Физическая сущность

    Свободным падением может называться равноускоренное движение тела в результате действующей на него силы тяжести, происходящее в вакууме. Атмосфера Земли способна тормозить ускорение и замедлять падающие предметы. Однако, если величина сопротивления воздуха небольшая, ей можно пренебречь. 2. Из-за воздействия центробежных сил на экваторе его значение немного меньше, а на полюсах, соответственно, больше.

    Величина ускорения свободного падения зависит от нескольких факторов:

    • географических координат, точнее, широты;
    • расстояния до поверхности планеты;
    • времени суток;
    • геомагнитных аномалий.

    Вектор свободного падения всегда направлен вниз. Это можно наглядно увидеть, подбросив какой-либо предмет. Благодаря воздействию ускорения, его движение будет постепенно замедляться. Затем оно полностью остановится и направится в обратную сторону.

    Формулы для расчёта

    Галилей понимал, что исследование падения тел с Пизанской башни является несовершенным. Был поставлен новый эксперимент, в котором учёному удалось увеличить время движения и уменьшить сопротивление воздуха. Отполированные латунные шарики скатывались по желобам, расположенным под определённым углом наклона. 2), где:

    • G — гравитационная постоянная;
    • M — масса планеты;
    • R — радиус планеты.

    При помощи этой зависимости можно рассчитать значение g на поверхности любой планеты во вселенной.

    Существуют задачи, для решения которых необходим более точный расчёт. В таком случае используется другая, расширенная формула: g=G (M/(R2+h)), ​где h — это высота над поверхностью планеты.

    Стоит помнить, что для максимальной точности расчётов придётся учитывать большое количество факторов. Ускорение может измеряться при помощи специального прибора — гравиметра.

    Ускорение на других планетах

    Как видно из формулы, гравитационное ускорение напрямую зависит от массы и радиуса планеты. Из этого следует, что значение g на других планетах будет отличаться от земного.

    Таблица показателя ускорения g для основных объектов Солнечной системы.

    НаименованиеУскорение, м/с. 11м/с. 2

    Воздействие перегрузок на человека

    Благодаря научно-техническому прогрессу и стремительному развитию технологий, современный человек имеет возможность пользоваться довольно быстрыми средствами передвижения. Чтобы попасть в любую точку планеты на самолёте, потребуется не более суток. Быстрая скорость передвижения неминуемо связана с таким понятием, как перегрузка.

    Любая перегрузка являет собой отношение двух ускорений:

    • негравитационного;
    • свободного падения.

    За единицу измерения принято брать гравитационное ускорение на Земле — 9,80665 м/с². Таким образом, нулевую перегрузку можно ощутить на себе лишь в невесомости.

    Перегрузка является векторной величиной. Для людей и других живых организмов огромное значение имеет её направление. Это связано с тем, что организм приспособлен к постоянному воздействию гравитационного ускорения.

    Характер положительной перегрузки заключается в том, что её вектор направлен вниз — от головы к ногам. Кровь оттекает от мозга и при показателе более 10 g человек может потерять сознание за считаные секунды. При отрицательном значении кровь, напротив, бьёт в голову. Это переносится гораздо хуже и может привести к кровоизлиянию и смерти.

    Показатель перегрузки для различных ситуаций:

    ПримерПоказатель, g
    Статичное положение,1
    Взлёт пассажирского авиалайнера1,5
    Приземление на парашюте1,8
    Раскрытие купола10−16
    Спуск космического аппарата «Союз»3−4
    Высший пилотаж на спортивном самолётеот -7 до +12
    Максимальная длительная перегрузка, переносимая человеком8−10
    Аварийный спуск из космоса20−26
    Рекордная не смертельная перегрузка при автокатастрофе214
    Торможение автоматического аппарата в атмосфере Венеры350
    Предел прочности твердотельного накопителя информации1500
    Снаряд в момент выстрела47 тыс.

    Военным и спортивным лётчикам приходится постоянно испытывать большие перегрузки. Для уменьшения вредного воздействия на организм существуют специальные защитные костюмы.

    Переносить перегрузку лучше всего лёжа на спине. Именно в таком положении находятся космонавты при взлёте ракет.

    Что такое ускорение? Виды ускорения. Формулы. Пример решения задачи :: SYL.ru

    Раздел физики, ответственный за изучение особенностей движения в пространстве тел, называется кинематикой. В данной статье рассмотрим, какие физические величины в кинематике используются для описания перемещений объектов, а также раскроем, что такое ускорение.

    Физические величины в кинематике

    Когда тело движется в пространстве, то нам важно знать, какое расстояние оно проходит за указанный промежуток времени и вдоль какой траектории движется.

    Для описания пройденных расстояний в физике используют понятие пути – L. В случае движения по окружности вместо пути пользуются понятием угла поворота – θ. Величину L в СИ измеряют в метрах (м), а величину θ – в радианах (рад.).

    Помимо пути важно знать также скорость движения тел. Под ней понимают быстроту прохождения расстояний. Математическое выражение для линейной скорости принимает вид:

    v¯ = d L / d t

    Для описания движения по окружности применяют угловую скорость ω, которая рассчитывается так:

    ω¯ = d θ / d t

    Третьей важной величиной кинематики является ускорение.

    Что такое ускорение? Это величина в физике, которая показывает, как быстро меняется скорость во времени. Математически это можно записать так:

    a¯ = d v¯ / d t

    Если подставить в эту формулу ускорения выражение для скорости, получим:

    a¯ = d2 L / d t2

    Ускорение – это первая производная скорости по времени или вторая производная по времени пройденного пути.

    Тангенциальное и нормальное ускорение

    Выше было дано определение, что такое ускорение. Оно называется полным. В общем случае направление полного ускорения не совпадает с направлением вектора скорости. Последний является касательной к траектории движения в любой ее точке.

    Поскольку скорость – это величина векторная, то ее изменение предполагает возможность менять модуль и направление. В первом случае говорят о наличии у тела тангенциального ускорения, во втором – нормального.

    Формула тангенциального ускорения at не отличается от таковой для ускорения полного a. Формула имеет вид:

    at = d v / d t

    То есть тангенциальное, или касательное, как его еще называют, ускорение является производной от модуля скорости по времени. Вектор at¯ совпадает с вектором v¯ при ускоренном движении и противоположен ему при замедленном движении.

    Нормальное ускорение – это физическая величина, которая приводит к искривлению прямолинейной траектории перемещения тел. Направлено оно вдоль радиуса кривизны траектории, то есть нормально по отношению к ней. Формула для его определения имеет вид:

    ac = v2 / r

    Нормальное ускорение ac зависит от модуля скорости v и радиуса кривизны траектории r. Очевидно, что в случае движения по прямой радиус r можно считать равным бесконечности. Последнее означает, что нормальное ускорение равно нулю для прямолинейного движения.

    Для движения по окружности вектор ac¯ направлен к ее центру вдоль радиуса. По этой причине величину ac также называют центростремительным ускорением.

    Полное ускорение

    Вектор полного ускорения – это всегда сумма тангенциальной и нормальной компонент. Поскольку они перпендикулярны друг другу, то для вычисления модуля полного ускорения можно воспользоваться теоремой Пифагора. Искомая формула ускорения полного примет вид:

    a = √(at2 + ac2)

    Чтобы определить, куда направлен вектор a¯, достаточно вычислить угол между ним и какой-либо компонентой. Например, угол φ между векторами a¯ и at¯ равен:

    φ = arctg(ac / at)

    Напомним, что центростремительное ускорение отлично от нуля только тогда, когда кривизна траектории движения отлична от бесконечности. В случае же прямолинейного движения полное ускорение по величине и направлению равно тангенциальной компоненте.

    Угловое ускорение

    Рассматривая, что такое ускорение, следует остановиться на соответствующей угловой характеристике.

    Выше было введено понятие угловой скорости, которая измеряется в радианах в секунду (рад/с). Если найти производную этой скорости по времени, то мы получим величину углового ускорения:

    α¯ = ω¯ / d t

    Несложно показать, что угловая величина связана с тангенциальной компонентой полного ускорения следующим соотношением:

    at = α × r

    При постоянном угловом ускорении касательная компонента at будет больше для точек, которые находятся дальше от оси вращения.

    К нормальной компоненте угловое ускорение не имеет никакого отношения.

    Решение задачи на определение ускорения

    Предположим, что, двигаясь с ускорением постоянным вдоль прямой линии, тело прошло расстояние 100 метров. Известно, что начальная скорость тела была равна 1 м/с. Отмеченное расстояние тело преодолело за 5,5 секунды. С каким ускорением происходило движение?

    Согласно условию задачи, речь идет о равноускоренном движении вдоль прямой траектории. Пройденный путь в этом случае может быть вычислен по формуле:

    L = v0 × t + a × t2 / 2

    Выражаем из равенства величину a, имеем:

    a = 2 × (L – v0 × t) / t2

    Все величины в правой части равенства известны из условия. Подставляем их и записываем ответ: a = 6,25 м/с2. То есть в течение каждой из 5,5 секунд скорость тела возрастает на 6,25 м/с. Найденное значение полного ускорения совпадает с тангенциальной компонентой.

    Объяснение урока: ускорение в зависимости от расстояния и времени

    В этом пояснении мы научимся рассчитывать ускорение используя начальную скорость объекта, его перемещение и его ускорение время по формуле 𝑠=𝑢𝑡+12𝑎𝑡.

    Представьте себе объект, движущийся по прямой линии со скоростью 𝑢. Предположим, что этот объект постоянно ускоряется так, что через некоторое время 𝑡, он движется с новой скоростью 𝑣.

    Если мы выберем начальный момент времени равным 0, то мы можем нанести точки для этого объекта на графике скорость-время следующим образом.

    Мы видим, что со временем скорость нашего объекта увеличивается. И нам это говорят увеличение происходит с постоянной скоростью. Следовательно, объект постоянно ускоряется, а так как ускорение вообще равно изменению скорости, деленному на соответствующее изменение во времени, мы можем написать 𝑎=𝑣−𝑢𝑡.

    Графически ускорение равно наклону прямой линии соединяя точки (0,𝑢) и (𝑡,𝑣).

    По этому графику мы можем определить полное перемещение нашего объекта, поскольку оно Ускоряться. Это смещение равно площади под кривой.

    Используя данную информацию о нашем объекте, мы можем вычислить его перемещение за этот промежуток времени. Чтобы вычислить площадь под нашей кривой, мы можем разделить это площадь на прямоугольную часть и треугольную часть следующим образом.

    Пусть переменная 𝑠 представляет смещение, мы пишем 𝑠=+.areaarea

    Поскольку площадь имеет форму треугольника, она будет равна половина произведения основания треугольника на его высоту. Треугольник основание — это время 𝑡, а его высота — это скорость 𝑣−𝑢: площадь=12×𝑡×(𝑣−𝑢).

    Площадь, будучи прямоугольником, будет равна основанию прямоугольника (𝑡) умножить на его высоту (𝑢): площадь=𝑡×𝑢.

    Таким образом, 𝑠=12×𝑡×(𝑣−𝑢)+𝑡×𝑢.

    Напоминая, что 𝑎=𝑣−𝑢𝑡, мы можем умножить обе части этого уравнения, чтобы получить 𝑎×𝑡=𝑣−𝑢.

    Следовательно, мы можем заменить 𝑣−𝑢 в нашем уравнении для смещение с 𝑎×𝑡 следующим образом: 𝑠=12×𝑡×(𝑎×𝑡)+𝑡×𝑢.

    Немного переставляя, получаем следующий результат.

    Формула: перемещение в терминах начальной скорости, прошедшего времени, и постоянное ускорение

    рассмотреть 𝑠=𝑢×𝑡+12×𝑎×𝑡.

    Этот результат представляет собой уравнение движения, описывающее смещение равномерно ускоряющийся объект с точки зрения его начальной скорости 𝑢, ускорение 𝑎 и ускорение время 𝑡.

    Обратите внимание, что это векторное уравнение. Это означает направления смещения, необходимо учитывать начальную скорость и ускорение.

    Эта формула применяется, даже если начальная скорость 𝑢 равно нулю. В этом случае график зависимости скорости от время появляется, как показано ниже, и соотношение для смещения 𝑠 упрощает до 𝑠=12×𝑎×𝑡(𝑢=0).

    Давайте потренируемся работать с общим уравнением движения, используя несколько Примеры.

    Пример 1: Анализ графика зависимости скорости от времени

    График показывает изменение скорости объекта во времени.

    1. Какова скорость объекта при 𝑡=0?
    2. Как долго объект ускоряется?
    3. Какова скорость объекта после его ускорения?
    4. Каково ускорение объекта?
    5. Каково перемещение объекта?

    Ответ

    Часть 1

    По горизонтальной оси времени линия 𝑡=0 перекрывает вертикальную ось (скорость). Показано, что скорость в это время 20 м/с.

    Часть 2

    Учитывая, как долго объект ускоряется, мы знаем, что это равно отрезок времени, в течение которого скорость объекта изменяется. На графике показано изменение скорости от 0 до 15 секунд, поэтому наш ответ состоит в том, что объект ускоряется за 15 секунд.

    Часть 3

    В конце этих 15 секунд, новая скорость объекта задается соответствующим значение вертикальной оси. Это значение 50 м/с.

    Часть 4

    Чтобы найти ускорение объекта, мы можем вспомнить, что в общем случае 𝑎=Δ𝑣Δ𝑡, где Δ𝑣 — изменение скорости и Δ𝑡 – соответствующее изменение во времени.

    Для рассматриваемого объекта, Δ𝑣=50/−20/=30/мсмсм а также Δ𝑡=15.с

    Следовательно, ускорение 𝑎=30/15=2/.mssms

    Часть 5

    Мы можем найти смещение объекта двумя разными способами.

    Во-первых, мы можем использовать алгебраический метод. Вспоминая уравнение движения 𝑠=𝑢×𝑡+12×𝑎×𝑡, где 𝑠 — перемещение объекта, 𝑢 — его начальная скорость, 𝑎 — его ускорение, а 𝑡 — время ускорения можно подставить в известные значения 𝑢, 𝑡 и 𝑎 следующим образом: 𝑠=(20/)×(15)+12×2/×(15)=300+225=525.mssmssmmm

    Другой способ определить это смещение — определить его графически. Смещение 𝑠 нашего объекта равно площади под кривая на нашем графике.

    На этом рисунке мы разделяем область под кривой на треугольную область (𝐴) и прямоугольная область (𝐴). Общая площадь (и, следовательно, смещение нашего объекта) определяется суммой 𝐴 и 𝐴: 𝑠=𝐴+𝐴.

    Поскольку фигура, соответствующая 𝐴, является треугольником, его площадь равна половине основания треугольника, умноженной на его высоту. Глядя на рисунок выше, мы видим, что база – это время 15 с а высота это скорость 50/−20/мс, или 30 м/с.

    Следовательно, 𝐴=12×(15)×(30/)=225.smsm

    Поскольку 𝐴 — прямоугольник, его площадь равна основанию (также 15 с) умножить на его высоту 20 м/с: 𝐴=(15)×(20/)=300.smsm

    Следовательно, 𝑠=225+300=525.ммм

    Перемещение объекта за этот интервал времени равно 525 м.

    Можно рассматривать ускорение на расстоянии, на котором начальная скорость и конечная скорость объекта известны, а также время, за которое объект ускоряется.

    Теперь рассмотрим пример с использованием этого метода.

    Пример 2. Определение смещения объекта, ускоряющегося в направлении начальной скорости

    Объект имеет начальную скорость 12 м/с. Объект ускоряется с 2,5 м/с 2 в том же направлении его скорости для время 1,5 с. Каково перемещение тела за это время? Отвечать до одного десятичного знака.

    Ответ

    Поскольку этот объект ускоряется с постоянной скоростью, мы можем описать его движение используя уравнения движения. В частности, мы будем использовать соотношение 𝑠=𝑢×𝑡+12×𝑎×𝑡, где 𝑠 — перемещение объекта, 𝑢 — его начальная скорость, 𝑎 — его ускорение, а 𝑡 — время его ускорения. Обратите внимание, что перемещение, скорость и ускорение являются векторами, указывая на то, что относительное важно направление этих величин.

    Наша постановка задачи говорит нам, что в данном случае все эти векторные величины имеют одинаковое направление. Моделирование объекта в виде точки, мы можем выбрать эти векторы с положительными величинами, чтобы указать на Правильно.

    Обратите внимание, что относительные длины трех стрелок нельзя сравнивать, так как они обозначают разные физические величины.

    На диаграмме выше показано, что перемещение объекта одинаково направление как его ускорение и начальная скорость. Поскольку эти значения положительным, то перемещение также будет положительным.

    Зная, что 𝑢=12/мс, 𝑎=2,5/мс, и 𝑡=1,5 с, 𝑠=(12/)×(1,5)+12×2,5/×(1,5)=18+2,8125=20,8125.mssmssmmm

    Округляя этот результат до одного десятичного знака, находим, что смещение объект 20,8 м.

    Пример 3. Определение расстояния, пройденного равномерно замедляющимся автомобилем

    Автомобиль движется на восток и проходит точку 𝑃 это 45 м к востоку от транспортной развязки. Когда автомобиль проходит точку 𝑃, его скорость 30 м/с и водитель притормаживает, ускоряясь на запад в 2,5 м/с 2 . какая расстояние к востоку от перекрестка находится автомобиль через 10 с после срабатывания тормозов применяемый?

    Ответ

    Эскиз автомобиля при прохождении точки 𝑃 может выглядеть как следует.

    В указанный момент водитель автомобиля нажимает на тормоз, давая автомобилю постоянное замедление 2,5 м/с 2 на запад, заставляя машину замедляться. После 10 секунд этого замедление, мы хотим найти общее расстояние автомобиля к востоку от пересекающиеся дороги.

    Поскольку замедление автомобиля постоянно во времени, мы можем описать движения автомобиля из точки 𝑃 по уравнению движение 𝑠=𝑢×𝑡+12×𝑎×𝑡, где 𝑠 — смещение автомобиля от точки 𝑃, 𝑢 — его начальная скорость, 𝑎 — его ускорение, а 𝑡 — прошедшее время.

    Это уравнение включает векторные величины, поэтому определяет положительное направление и отрицательное направление в нашем сценарии будут полезный. Мы выбираем положительное направление на восток, т. е. любое вектор, указывающий на запад, считается отрицательным. Следовательно начальная скорость автомобиля 𝑢 положителен, а его ускорение 𝑎 отрицательный.

    Решение смещения автомобиля от точки 𝑃 согласно приведенному выше уравнению движения имеем 𝑠=(30/)×(10)+12×−2,5/×(10)=300+(−125)=175.mssmssmmm

    Здесь нужно вспомнить, что машина стартовала 45 м к востоку от с. пересекающиеся дороги. Таким образом, общее расстояние автомобиля к востоку от Перекресток десять секунд после прохождения точки 𝑃 175+45=220.mmm

    Теперь рассмотрим ситуацию, в которой движущийся объект ускоряется в направление, противоположное его начальному движению, заканчивающееся конечной скоростью и смещение также противоположно направлению начальной скорости.

    Пример 4: Расчет смещения объекта в направлении движения объекта Начальная скорость

    Объект имеет начальную скорость 32 м/с. Объект ускоряется с 12 м/с 2 в направлении, противоположном его скорости, за время 5,5 с. Каково чистое смещение объекта в направлении его первоначального скорость за это время?

    Ответ

    В этом примере используются векторные величины, поэтому важно соблюдать осторожность. отслеживать положительные и отрицательные знаки. Мы можем произвольно выбрать начальная скорость объекта должна быть направлена ​​вправо, а это означает, что его ускорение направлено влево следующим образом.

    На этой диаграмме мы не можем сравнивать относительную длину стрелок потому что они соответствуют разным физическим величинам. мы знаем однако чистое смещение объекта будет положительным, если оно указывает вправо относительно начальной точки и отрицательный, если он указывает Слева.

    Поскольку наш объект ускоряется с постоянной скоростью, мы можем использовать следующее уравнение движения для описания его движения: 𝑠=𝑢×𝑡+12×𝑎×𝑡, где 𝑠 — перемещение объекта, 𝑢 — его начальная скорость, 𝑎 — его ускорение, а 𝑡 — время, за которое объект ускоряется.

    Мы определили начальную скорость 𝑢 быть позитивным (+32/) мс, поэтому ускорение 𝑎 должно быть отрицательным (−12 м/с). Подставляем эти значения в наше уравнение вместе со временем 𝑡=5,5 с, получаем 𝑠=(32/)×(5,5)+12×−12/×(5,5)=176+(−181,5)=−5,5.mssmssmmm

    Чистое смещение объекта в направлении его начального скорость −5,5 м, сообщая нам, что стрелка чистого смещения на рисунке выше должна указывать на слева.

    Наконец, мы рассмотрим пример, в котором мы находим скорость объекта.

    Пример 5: Расчет начальной скорости в направлении ускорения

    Автомобиль, который первоначально двигался с постоянной скоростью, имеет чистое перемещение 45 м после ускорение по прямой на 1,5 м/с 2 в течение 15 секунд. Какова была начальная скорость автомобиля в направлении ускорение?

    Ответ

    Поскольку перемещение, ускорение и скорость являются векторными величинами, мы должны быть осторожны, чтобы понять относительные направления, в которых они действуют.

    Обратите внимание, что данный рабочий объем (45 м) и ускорение (1,5 м/с 2 ) оба положительны, что говорит нам о том, что эти векторы указывают в одном направлении. Мы хотим найти начальную скорость автомобиля относительно знака принято считать, что движение в направлении ускорения автомобиля положительный.

    Так как автомобиль ускоряется с постоянной скоростью в течение временной интервал 15 с, мы можем использовать следующее уравнение движения, чтобы описать его движение: 𝑠=𝑢×𝑡+12×𝑎×𝑡, где 𝑠 – водоизмещение автомобиля, 𝑢 — его начальная скорость, 𝑎 — его ускорение, а 𝑡 – время ускорения.

    Мы можем изменить это уравнение, чтобы найти начальную скорость 𝑢. Вычитание 12×𝑎×𝑡 с обеих сторон дает 𝑠−12×𝑎×𝑡=𝑢×𝑡.

    Деление обеих частей уравнения на 𝑡 дает 𝑠−×𝑎×𝑡𝑡=𝑢 чтобы 𝑢=𝑠−×𝑎×𝑡𝑡.

    Подставляя известные значения для 𝑠 (45 м), 𝑎 (1,5 м/с 2 ), и 𝑡 (15 с), имеем 𝑢=(45)−×1,5/×(15)15=−8,25/.mmsssms

    Тот факт, что 𝑢 отрицательно, означает, что он действует в направлении против ускорения автомобиля. Относительно этого положительного направления тогда начальная скорость автомобиля −8,25 м/с.

    Ключевые моменты

    • Уравнение движения 𝑠=𝑢×𝑡+12×𝑎×𝑡 может быть получен алгебраически или из графика движения объекта при постоянное ускорение.
    • Величины 𝑠 (смещение), 𝑢 (начальная скорость) и 𝑎 (ускорение) в этом уравнении все векторы и поэтому могут быть положительными или отрицательными.
    • Это уравнение можно составить алгебраически так, что любая из переменных (𝑠, 𝑢, 𝑎, или 𝑡) является подлежащим.

    5 фактов, которые вы должны знать — Lambda Geeks

    Ускорение — это скорость изменения скорости. Поскольку скорость является векторной величиной, ускорение также является векторной величиной. В результате она требует как величины, так и направления. Итак, в этой статье мы рассмотрим, как найти величину ускорения.

    Мы используем формулы и соотношения для расчета величины. Мы можем представить величину в терминах единиц. Поскольку ускорение определяется как изменение скорости во времени, единицей СИ для него является 9.0219 ㎨ . На ускорение влияют различные факторы, такие как скорость, время, сила и так далее. Мы рассмотрим несколько различных методов оценки величины ускорения. Давайте читать дальше.

    1. Как найти величину ускорения Из определения ускорения:

     Как мы все знаем, ускорение относится к скорости изменения скорости. Если начальная скорость тела равна vi, а его конечная скорость равна vf, ускорение можно рассчитать, разделив изменение скорости на интервал времени Δt:

    (Здесь векторная величина выделена жирным шрифтом, а вертикальные линии обозначают величину вектора или мы можем сказать абсолютное значение вектора, которое всегда положительно.)

    Используя скорость и время, приведенное выше уравнение может быть использовано для определения величины ускорения.

    2. Как найти величину ускорения Из второго закона Ньютона:

    Второй закон Ньютона гласит, что сила получается путем умножения ускорения на массу тела. Так как же определить величину ускорения?

    Итак, согласно второму закону Ньютона сила, действующая на тело, пропорциональна его ускорению, тогда как масса обратно пропорциональна ускорению . Переведем эти утверждения в формулу величины ускорения:

    и

    Таким образом,

    3. Как найти величину ускорения Из векторных составляющих ускорения:

    Ускорение является векторной величиной, как мы все знаем. Чтобы получить эту величину, сложите вместе компоненты ускорения. Здесь можно использовать простое правило сложения векторов. Если задействованы две компоненты вектора, мы можем написать:

    В декартовой плоскости мы можем использовать координаты X и Y. Как в этом случае определить величину ускорения? В декартовой системе координат компоненты X и Y перпендикулярны друг другу. Величину ускорения можно рассчитать, возведя значения в квадрат, а затем вычислив квадратный корень из суммы.

    В результате получается следующее уравнение:

    Формула величины ускорения в трехмерном пространстве:

    4. Как найти величину центростремительного ускорения:

    Из-за постоянного изменения направления при круговом движении скорость меняется, что приводит к ускорению. Ускорение направлено в направлении центра круга. Возведение скорости тела v в квадрат и деление ее на расстояние тела от центра окружности дает величину центростремительного ускорения. Таким образом, центростремительное ускорение:

    5. Как найти величину ускорения Из уравнений движения:

    Уравнения движения — это, по сути, уравнения, которые объясняют движение любой физической системы и демонстрируют взаимосвязь между перемещением объекта, скоростью, ускорением и временем.

    Когда величина ускорения постоянна, кинематическое уравнение движения в одном измерении также используется для расчета величины ускорения.

    Ниже приведены уравнения движения:

    Когда мы делаем ускорение предметом уравнения, мы получаем следующее:

    •   (Это то же самое, что мы получили из определения ускорения.)

    Вот как мы можем найти величина ускорения.

    Решенные примеры Нахождение величины ускорения:

    Задача 1:

    Автомобиль трогается с места и достигает скорости 54 км/ч за 3 секунды. Найдите его ускорение?

    Решение: Автомобиль трогается с места. Таким образом, начальная скорость автомобиля

    VI = 0 м/с

    VF = 54 км/ч

    = (54 ✕ 1000)/3600

    = 15 м/с

    ΔT = 3 с

    Таким Ускорение : A = (VF – VI) / ΔT

    = (15 – 0) / 3

    =

    Задача 2:

    Определить удручение применяется к объекту массой 3 кг, а затем к объекту массой 6 кг.

    Решение: Применяемая сила F = 12 N

    Массы объекта M1 = 3 кг

    M2 = 6 кг

    Ускорение объекта с массой 3 кг

    a1 = F / M1

    = 12 = 12 кг

    a1 = F / M1

    = 12 = 12 = 12. / 3

    =

    Ускорение объекта с массой 6 кг

    A2 = F / M2

    = 12/6

    =

    Потому что масса и аккурация. что по мере увеличения массы ускорение уменьшается.

    Задача 3:

    Тело движется вдоль оси x согласно соотношению 

    , где x выражено в метрах, а t – в секундах. Найдите ускорение тела в момент времени t = 3 с.

    Solution: Here :

                              t  = 3s

    Velocity v = dX/dt

                     = d/dt ()

                     = -2 + 6t

    Acceleration : a = dv/dt

                              = d/dt (-2 + 6t)

    =

    Как мы видим, для этого движения ускорение не зависит от времени; ускорение будет постоянным на протяжении всего движения, а величина ускорения составит .

    Задача 4:

    Вычислите центростремительное ускорение точки на расстоянии 7,50 см от оси ультрацентрифуги, вращающейся со скоростью  об/мин.

    Решение: Здесь мы даем:

    Расстояние от центра R = 7,5 см

    = 0,0750 м

    Англосная скорость

    Сейчас

    = 589 м/с

    Таким образом, Centripetal Acceler AcceLater Acceleratore Acceleratore AcceLatorpatore AcceLatorptatore AcceLatoreTATATATTAL ACCELATTAL ACCELATTAL ACCELATTAL ACCELATTAL ACCELATTAL ACCELATTAL ACCELATTAL ACCELARETAL ACCELATTO

    =

    =

    Мгновенное ускорение: определение, формула и многое другое

    В этой статье вы узнаете, что мы подразумеваем под мгновенным ускорением или проще ускорением при описании движения частицы.

    Мы рассмотрим определение и формулу мгновенного ускорения на примере, который демонстрирует, как использовать формулу на практике.

    Мы также рассмотрим другие важные вещи, которые вам следует знать, например, как представлено мгновенное ускорение на графике скорость-время .

    Определение и формула для мгновенного ускорения

    Ускорение a, которое имеет частица в момент t, равно значению, к которому среднее ускорение, рассчитанное для интервала времени Δt, включающего момент t, приближается как интервал времени Δt становится все меньше и меньше, т. е. по мере приближения Δt к 0.

    Мы знаем, что среднее ускорение a за интервал времени Δt выражается как:

    a = Δv
    Δt

    где Δv — изменение скорости, происходящее в течение Δt.

    Формула для мгновенного ускорения a почти такая же, за исключением того, что нам нужно указать, что нас интересует, к чему приближается отношение Δv к Δt, когда Δt приближается к нулю.

    Мы можем указать это, используя обозначение предела.

    Итак, формула мгновенного ускорения:

    a = lim   Δv
    Δt→0 Δt

    Чтобы продемонстрировать, как использовать эту формулу на практике, рассмотрим простой пример.

    Рассмотрим частицу, скорость которой (в метрах в секунду) в момент времени t (в секундах) равна 2t 2 :

    v = 2t 2

    Итак, в 1 с скорость равна 2 м /с, через 2 с скорость 8 м/с, через 3 с скорость 18 м/с и т. д.

    Допустим, мы хотим найти ускорение частицы в момент времени t = 3 с.

    Первое, что нам нужно сделать, это выбрать интервал времени Δt, который включает момент 3 с.

    Этот интервал времени Δt начинается в некоторый момент t 1 и заканчивается в некоторый момент t 2 так, что

    t 1 ≤ 3 с ≤ t 2

    1  = 3 с так, чтобы t 1 было как можно ближе к 3 с, а Δt можно было уменьшить, выбрав значения t 2 , которые ближе к 3 с.

    t 1 = 3 с

    t 2 > 3 с

    Начнем с выбора t 2 равным 3,1 с.

    T 1 = 3 S

    T 2 = 3,1 S

    ΔT = T 2 – T 1 = 3,1 S – 3 с = 0,1 с

    . :

    а = Δv  = v 2 − v 1
    Δt t 2 − t 1

    Let’s find the velocity v 1 at instant t 1 :

    v 1 = 2t 1 2

    V 1 = 2 (3) 2 м/с

    V 1 = 18 м/с

    и скорость v 2 на мгновенном T 2 :

    V

    0 2

    . = 2t 2 2

    v 2 = 2 (3,1) 2 м/с

    V 2 = 19,22 м/с

    Теперь мы можем рассчитать среднее ускорение:

    A = V. выражение приближается к 4t.

    Следовательно, мгновенное ускорение a равно 4t:

    a = 4t

    Итак, ускорение частицы в любой момент времени t равно 4t.

    Таким образом, в момент t = 3 с ускорение равно 4 × 3 м/с 2 , т. е. 12 м/с 2 .

    Instantaneous acceleration as derivative

    The limit

    A = V
    A = v

    V

    A =

    . v 1

    A =
    t 2 − t 1

    a =  19.22 m/s − 18 m/s
    3.1 s − 3 s

    а = 1,22 м/с
    0,1 S

    A = 12,2 м/с 2

    Таким образом, когда интервал времени ΔT составляет от 3 с и 3,1 с, среднее удручение составляет 12,2 м/с 2 .

    Посмотрим, что произойдет, если мы выберем меньший интервал времени Δt, где t 2 равно 3,01 с.

    t 1 = 3 с

    t 2 = 3,01 с

    Δt = 3,01 с − 3 с = 0,01 с

    Мы уже знаем, что скорость v 1 в момент времени t 1 составляет 18 м/с.

    Скорость v 2 в мгновенном T 2 IS:

    V 2 = 2T 2 2

    V 2 = 2 (3.017

    9000 2 2 2 2 2 2 2

    V 2 = 2 (3,0117 2

    V 2 = 2 (3.0117 2

    V 2 = 2

    V 2 = 2

    V 2 = 2
    . 2 = 18.1202 m/s

    The average acceleration is:

    a =  v 2 − v 1
    t 2 − t 1

    a =  18.1202 m/s − 18 m/s
    3.01 s − 3 s

    a =  0. 1202 m/s
    0.01 s

    a = 12,02 м/с 2

    Таким образом, когда интервал времени Δt находится между 3 с и 3,01 с, среднее ускорение составляет 12,02 м/с 2 .

    В последний раз выберем еще меньшее Δt, с t 2 равно 3,001 с.

    t 1 = 3 s

    t 2 = 3.001 s

    Δt = 0.001 s

    The velocity v 2 at instant t 2 is:

    v 2 = 2t 2 2

    v 2 = 2 (3.001) 2 m/s

    v 2 = 18.012002 m/s

    The average acceleration is:

    a =  18.012002 m /с − 18 м/с
    3.001 s − 3 s

    a =  0.012002 m/s
    0.001 s

    a = 12.002 m/s 2

    So, as Δt is становится все меньше и меньше, среднее ускорение приближается к 12 м/с 2 .

    Мы можем продолжать выбирать все меньше и меньше Δt до бесконечности и все ближе и ближе приближаться к 12 м/с 2 .

    Однако мы можем показать, что среднее ускорение приближается к 12 м/с 2 , по мере того, как Δt становится все меньше и меньше, более строгим образом, чтобы мы могли быть уверены, что ускорение в момент 3 с равно 12 м/с 2 .

    Прежде всего, мы будем ссылаться на момент 3 s как на t, чтобы показать, что то, что мы собираемся сделать, относится не только к конкретному моменту 3 s, но и к любому моменту t.

    Начнем, как и раньше, с рассмотрения интервала времени Δt, который начинается в некоторый момент t 1 и заканчивается в некоторый момент t 2 , такой, что

    t 1 ≤ t ≤ t 2

    Снова выберем t 1  = t так, чтобы t 1 было как можно ближе к t, а Δt можно уменьшить, выбрав значения t 2 которые ближе к t.

    T 1 = T

    T 2 > T

    , так как ΔT = T 2 – T 1 и T 1 = T, мы можем написать:

    T 1 = T, мы можем написать:

    T 1 = T. − т

    т 2 = t + Δt

    Это означает, что границами интервала времени Δt являются момент t и момент t + Δt.

    Скорость v T в мгновенном T IS:

    V T = 2T 2

    , а скорость V T+ΔT в Instant T+ΔT IS:

    902 V

    4444444444444444444444444441 в instant t+ΔT:

    902 v

    4444444444444444444444444444441 в instant t+ΔT:

    902 v

    4444444444444444444444444440 = 2 (t + Δt) 2

    v t+Δt = 2 (t 2 + 2tΔt + Δt 2 )

    2

    1719 t 0118 + 4TΔT + 2ΔT 2

    Теперь мы можем рассчитать среднее ускорение для ΔT:

    A = V T + Δ – V 0 T98444444444444444444444444444444448944894489848984444444444444444444444444444444444444444444444444444444444444444444444444444444444444444444444444444444444444489н.

    a =  2t 2 + 4tΔt + 2Δt 2 − 2t 2
    Δt

    a =  4tΔt + 2Δt 2
    Δt

    a = 4t + 2Δt

    Обратите внимание, что это выражение, 4t + 2Δt, объясняет, почему среднее ускорение, которое мы вычисляли вручную ранее, составляло 12,2 м/с 2

    , когда Δt равнялось 1,1 с, 1,1 с м/с 2 , когда Δt равно 0,01 с, и 12,002 м/с 2 , когда Δt равно 0,001 с:

    (4 × 3 + 2 × 0,1) м/с 2 = 12,1 м/1 7 90

    (4 × 3 + 2 × 0,01) м/с 2 = 12,02 м/с 2

    (4 × 3 + 2 × 0,001) м/с 2 = 12,002 м/с 2

    Теперь вспомним, что мгновенное ускорение равно значению, к которому приближается среднее ускорение по мере приближения интервала времени Δt к 0:

    a = 5lim6   Δv
    Δt→0 Δt

    a =  lim   v t+Δt − v t
    Δt→0 Δt

    В этом случае мы обнаружили, что среднее ускорение равно 4t + 2Δt. SO,

    A = LIM 4T + 2ΔT
    ΔT → 0

    AST → 0

    AST → 0

    AST → 0

    lim   v t+Δt − v t
    Δt→0 Δt

    is called the derivative v относительно t, который записывается как

    Следовательно, мы говорим, что мгновенное ускорение есть производная скорости по времени:

    a =  dv
    dt
    90, так как скорость есть производная от положения по времени:

    v = dx
    dt

    Мгновенное ускорение можно записать как:0483

    a =  dv  =  d   dx  =  d 2 x
    dt dt dt dt 2

    Таким образом, мгновенное ускорение есть вторая производная положения по времени:

    a = d 2 x
    dt 2 9488850003

    График зависимости ускорения от времени

    Часто, чтобы показать, как ускорение частицы изменяется во времени, используется график зависимости ускорения от времени .

    Вот как может выглядеть график зависимости ускорения от времени для движущейся частицы:

    График зависимости ускорения от времени; ускорение в момент времени 0 равно 0, затем становится положительным и, наконец, через 9 секунд возвращается обратно к 0,a210864t(s)897654321O2)(м/с

    ). имеет ускорение a = 6 м/с 2 :

    График зависимости ускорения от времени; в 4 секунды ускорение равно 6 метрам в секунду в квадрате.

    Давайте покажем, как ускорение этой частицы изменяется во времени, с помощью графика зависимости ускорения от времени

    График функции 4t представляет собой линию, поэтому нам нужно найти только две точки, чтобы нарисовать ее

    Чтобы не усложнять расчеты , выберем момент t 1 = 0 с и момент времени t 2 = 1 с:

    t 1 = 0 с, a 1 = (42 × 0) м/с 2 t 2 4 = 3 0 900 = 1 с, a 2 = (4 × 1) м/с 2 = 4 м/с 2

    Теперь, когда мы нашли две точки, мы готовы нарисовать график зависимости ускорения от времени:

    График зависимости ускорения от времени, состоящий из линии, проходящей через точки (0, 0) и (1, 4). a210864t(s)321O2)(m/s

    Графики зависимости скорости от времени и мгновенного ускорения

    Мгновенное ускорение как наклон касательной к графику зависимости скорости от времени

    Рассмотрим график зависимости скорости от времени для движения частицы:

    График зависимости скорости от времени; скорость в момент времени 0 равна 0, затем становится положительной и, наконец, возвращается к 0.vtO

    Помните, что график зависимости скорости от времени показывает, как скорость частицы изменяется во времени.

    Как мы видели ранее, когда мы хотим найти ускорение частицы в момент t, мы рассматриваем интервал времени Δt, который начинается в t и заканчивается в t + Δt:

    График зависимости скорости от времени; в момент t скорость v меньше t; в момент t + Δt скорость равна v sub-t + Δt; Указаны Δv и Δt. vtOtΔt+tttΔt+ΔtΔvvv

    Ускорение в момент t равно любому среднему ускорению для Δt, приближающемуся по мере приближения Δt к нулю.

    Мы можем использовать эту информацию, чтобы определить, чем представлено мгновенное ускорение на графике зависимости скорости от времени.

    Сначала проведем секущую, проходящую через точки t и t + Δt на графике:

    График зависимости скорости от времени; секущая линия, проходящая через точки t и t + Δt на графике.vtOttΔt+ΔttΔt+tΔvvv

    и наклон секущей.

    Таким образом, ускорение в момент t равно тому, к чему приближается наклон секущей при приближении Δt к нулю.

    По мере того, как Δt становится все меньше и меньше, секущая линия все ближе и ближе к линии, касательной к графику в точке t:

    График зависимости скорости от времени; когда Δt приближается к 0, секущая линия приближается к касательной в точке t на графике. точка т.

    Следовательно, ускорение в момент времени t равно наклону линии, касательной к графику зависимости скорости от времени в точке t.

    График зависимости скорости от времени; линия, касательная к графику в точке t.vtOt

    . Если обозначить наклон касательной через m T , then we can write

    a =  dv  = m T
    dt

    How to determine the sign of the instantaneous acceleration from a velocity vs time graph

    Определить знак мгновенного ускорения по графику зависимости скорости от времени довольно просто.

    Чтобы показать, как это сделать, давайте вернемся к тому же графику зависимости скорости от времени, который мы видели раньше, и рассмотрим момент времени t, в который мы хотим узнать знак ускорения:

    График зависимости скорости от времени; в момент t скорость v sub-t.vtOttv

    Мы уже знаем, что ускорение в момент t равно наклону линии, касательной к графику в точке t.

    Давайте изобразим касательную и назовем θ углом, который она образует с положительной осью t:

    График зависимости скорости от времени; угол θ, который образует касательная в точке t с положительной осью t. .

    Когда угол θ положительный:

    График зависимости скорости от времени; угол θ, который касательная образует с положительной осью t, положителен. vtOttvθ

    Наклон касательной положителен, и поэтому мгновенное ускорение положительно.

    Когда угол θ отрицателен:

    График зависимости скорости от времени; угол θ, который касательная образует с положительной осью t, отрицателен. vtOttvθ

    Наклон касательной отрицателен, поэтому мгновенное ускорение отрицательно.

    Наконец, когда угол θ равен нулю:

    График зависимости скорости от времени; касательная является горизонтальной линией. vtOttv

    Наклон касательной равен нулю, поэтому мгновенное ускорение равно нулю.

    Таким образом, знак ускорения в момент времени t совпадает со знаком угла θ, который линия, касательная графика зависимости скорости от времени в точке t, образует с положительной осью t.

    Итак, мы можем легко определить, когда ускорение положительное, отрицательное и нулевое, просто взглянув на угол θ в разных точках на графике зависимости скорости от времени:

    График зависимости скорости от времени; сначала ускорение положительно, затем на мгновение становится равным нулю и, наконец, становится отрицательным. vtOa0>a0 Что такое замедление?

    Когда ускорение вызывает уменьшение скорости по величине, это иногда называют замедлением .

    Итак, когда частица имеет положительную скорость и отрицательное ускорение или отрицательную скорость и положительное ускорение, вы можете услышать, что ускорение частицы называется замедлением.

    Вы также можете прочитать:

    • Движение с постоянным ускорением вдоль прямой линии
    • Ускорение свободного падения вблизи поверхности Земли
    • Средняя скорость: определение, формула, примеры и т. д.
    • Равномерное линейное движение: постоянная скорость Движение по линии

    Ускорение: определение, формула, примеры, виды

    Ускорение в нашем повседневном разговоре означает ускорение. Это может происходить в различных приложениях, будь то транспорт, человек, животное, объект, оборудование и т. д. Изменение скорости объекта называется ускорением. Вот почему чем больше ускорение, тем больше изменение скорости за данное время

    Сегодня вы познакомитесь с определением, примерами, формулой, единицей измерения, типами, вычислениями и разницей между ускорением и скоростью. Вы также изучите график зависимости скорости от времени.

  • 2 Формула и единица ускорения
    • 2.1 Подпишитесь на нашу рассылку
  • 3 Примеры
  • 4 Типы ускорения
    • 4. 1 Равномерное и неравномерное ускорение:
    • 4,2 Среднее ускорение:
      • 4.2.1 Посмотрите видео ниже, чтобы узнать больше о ускорении:
  • 5 Расчеты
      • 5.0,1 Пример 1
      • 5.0.2 Пример 2
      • 5.0.1 5.
      • 5.0.2. Пример 3
      • 5.0.4 Пример 4
    • 5.1 Пожалуйста, поделитесь!
  • Что такое ускорение?

    Ускорение относится к скорости изменения скорости во времени или скорости, с которой скорость меняется во времени. Это происходит с точки зрения скорости и направления. когда точка или объект движется по прямой линии, ускоряется, если он ускоряется или замедляется. Движение по окружности ускорено, даже если скорость постоянна. Следовательно, направление постоянно меняется, этот эффект также способствует ускорению всех других видов движения.

    Поскольку ускорение имеет как величину, так и направление, оно является векторной величиной. При этом ускорение можно определить как изменение вектора скорости в интервале времени, деленное на интервал времени. Мгновенное ускорение, известное как точное движение и местоположение, определяется предельным отношением изменения скорости в течение заданного интервала времени к интервалу времени, если интервал времени стремится к нулю.

    Например, если скорость выражена в метрах в секунду, ускорение будет выражено в метрах в секунду в квадрате.

    Самое запутанное в ускорении и скорости то, что это не одно и то же. Большинство людей думают, что если объект имеет большую скорость, то и ускорение тоже будет большим. Кроме того, подумайте о том, что скорость объекта мала, тогда ускорение мало. Но это не так. Значение скорости в данный момент не определяет ускорение. То есть человек может изменять свою скорость с высокой скоростью, независимо от того, движется ли он медленно или быстро.

    Подробнее: Расстояние и смещение

    Формула и единица измерения ускорения

    Формула ускорения задается следующим образом:

    Ускорение равно конечной скорости минус начальная скорость во времени.

    Тогда ускорение равно изменению скорости во времени. Математически это можно выразить следующим образом:

    Вы можете использовать уравнение ускорения для расчета ускорения. Вот наиболее распространенная формула ускорения:

    a=

    , где Δv — изменение скорости, а Δt — изменение времени.

    Вы также можете записать уравнение ускорения следующим образом:

    a=

    v(f)−v(i)
    t(f)−t(i)

    В этом уравнении ускорения v(f) — конечная скорость, а v(i) — начальная скорость. T(f) — конечное время, а t(i) — начальное время.

    Подробнее: Как сила изменяет состояние движения

    где:

    Подпишитесь на нашу рассылку новостей

    a – ускорение в м.с -2

    V f – конечная скорость в мс -1

    t – интервал времени в с и

    – малое изменение скорости в мс -1

    3 Итак, Единицей ускорения в системе СИ может быть метр в секунду м/с 2

    Примеры

    Ниже представлена ​​распространенная форма ускорения, которая встречается в нашей повседневной жизни.

    • Мальчик бежит с разной скоростью
    • Движение пчелы в другом направлении
    • Кто-то идет
    • Движущееся транспортное средство
    • Перемещение тележки в магазине глоссария
    • Движение стрелки
    • Мяч, отбитый ногой
    • Некоторые летающие птицы
    • Движение акулы с постоянной скоростью.

    Подробнее: Связь между силой и движением

    Типы ускорения

    Различные типы ускорения: равномерное и неравномерное ускорение и среднее ускорение.

    Равномерное и неравномерное ускорение:

    В ситуации, когда скорость остается постоянной, но тело ускоряется, возможен круговой, когда скорость остается постоянной, но поскольку направление меняется, следовательно, изменяется и скорость. При этом говорят, что тело ускоряется.

    Среднее ускорение:

    Этот тип ускорения за некоторое время определяется как общее изменение скорости в заданном интервале, деленное на время, затраченное на изменение. Для данного интервала времени его можно обозначить как  . математически,

    Посмотрите видео ниже, чтобы узнать больше об ускорении:

    Подробнее: Как сила изменяет состояние движения

    Расчеты

    Давайте рассмотрим несколько примеров, чтобы увидеть, как выглядит ускорение.

    Пример 1

    Белоголовый орлан летит влево со скоростью 34 метра в секунду, когда порыв ветра дует обратно на орла, заставляя его замедляться с согласованным ускорением величиной 8 метров в секунду в квадрате. Какова будет скорость белоголового орлана после того, как ветер будет дуть в течение 3 секунд?

    Ответ

    Из нашего уравнения

    Мы производим v F Предметом Формулы

    V F = V I + AT

    V F = -3441 + M0003

    V F = -3441 + 80003

    V F = –3441 +. м/с 2

    V f = -34м/с + 8м/с 2 (3с)

    V f = -10м/с

    Итак, конечная скорость = +10м/с s

    Пример 2

    Предположим, что автомобиль движется от скорости 20 м/с до конечной скорости 30 м/с за 5 секунд, рассчитайте его среднее ускорение.

    Answer

    Using a = v – u/t

    V = 30m/s

    U = 20m/s

    T = 5seconds

     a = 30 – 20/5

     a = 10/5

     a = 2 м/с 2

    Ускорение является векторной величиной, а векторная величина имеет как величину, так и направление. если объект меняет свою скорость, это показывает, что такой объект ускоряется. Например, велосипед движется со скоростью 2 м/с, а автомобиль движется со скоростью 5 м/с.

    Подробнее:

    Пример 3

    Тигровая акула-невротик начинает движение из состояния покоя и равномерно разгоняется до 12 метров в секунду за 3 секунды. Какова была величина среднего ускорения тигровой акулы?

    a = 4 м/с 2

    Пример 4

    Каково будет ускорение тела, движущегося с постоянной скоростью?

    Ответ

    Учитывая, что скорость постоянна, поэтому начальная и конечная скорости равны и даны как V. следовательно, ускорение дано как:

    a =  a =   следовательно, a = 0

    Это все для этой статьи, где обсуждаются определение, примеры, формула, единица измерения, типы и расчеты по ускорению. Я надеюсь, что вы многое узнали из чтения, если да, поделитесь с другими студентами. Спасибо за чтение, увидимся!

    Постоянное ускорение: определение, примеры и формула

    Постоянное ускорение: определение, примеры и формула

    Выберите язык

    Предлагаемые вам языки:

    Европа

    английский (DE) английский (Великобритания)

    StudySmarter — универсальное учебное приложение.

    4. 8 • Рейтинг +11k

    Более 3 миллионов загрузок

    Бесплатно

    Сохранять

    Распечатать

    Редактировать

    Постоянное ускорение

    Содержание :

    ОГЛАВЛЕНИЕ

      Ускорение определяется как изменение скорости во времени. Если скорость изменения скорости тела остается постоянной с течением времени, это известно как постоянное ускорение .

      Мяч, брошенный с высоты, свободно падающий под действием силы тяжести без какой-либо внешней силы, действующей на него, будет падать с постоянным ускорением, равным ускорению свободного падения.

      В действительности очень сложно реализовать идеальное постоянное ускорение. Это связано с тем, что на объект всегда будет действовать несколько сил. В приведенном выше примере на мяч также будут действовать различные атмосферные силы, такие как сопротивление воздуха. Однако изменения в результирующем ускорении могут быть достаточно малы, чтобы мы могли моделировать его движение, используя концепции постоянного ускорения.

      Графики постоянного ускорения

      Можно графически представить движение объекта. В этом разделе мы рассмотрим два типа графиков, которые обычно используются для представления движения объекта, движущегося с постоянным ускорением:

      1. Графики смещения-времени

      2. Графики скорости-времени

      Графики смещения-времени

      Движение объекта можно представить с помощью графика смещения-времени.

      Перемещение представлено по оси Y, а время (t) — по оси X. Это означает, что изменение положения объекта отображается в зависимости от времени, необходимого для достижения этого положения.

      Вот несколько моментов, о которых следует помнить при построении графиков смещения и времени:

      • Поскольку скорость — это скорость изменения смещения, градиент в любой точке дает мгновенную скорость в этой точке.

      • Средняя скорость = (полное перемещение)/(затраченное время)

      • Если график перемещение-время представляет собой прямую линию, то скорость постоянна, а ускорение равно 0.

      Следующее перемещение- временной график представляет собой тело с постоянной скоростью, где s представляет перемещение, а t время, затраченное на это перемещение.

      График перемещения-время для тела, движущегося с постоянной скоростью, Нилабро Датта, Study Smarter Originals

      Следующий график смещения-время представляет стационарный объект с нулевой скоростью.

      График перемещения-время для тела с нулевой скоростью, Нилабхро Датта, Study Smarter Originals

      Следующий график смещения-время представляет объект, движущийся с постоянным ускорением.

      График перемещение-время для тела, движущегося с постоянным ускорением, Нилабхро Датта, Study Smarter Originals

      Графики зависимости скорости от времени

      Движение объекта также может быть представлено с помощью графика зависимости скорости от времени. Обычно скорость (v) представлена ​​по оси Y, а время (t) по оси X.

      Вот несколько моментов, которые следует учитывать при построении графиков скорости-времени:

      • Поскольку ускорение — это скорость изменения скорости, на графике скорость-время градиент в точке дает ускорение объекта в этой точке. точка.

      • Если график зависимости скорости от времени представляет собой прямую линию, то ускорение постоянно.

      • Область, ограниченная графиком скорость-время и осью времени (горизонтальная ось), представляет расстояние, пройденное объектом.

      • Если движение происходит по прямой линии с положительной скоростью, то площадь, ограниченная графиком скорость-время и осью времени, также представляет собой перемещение объекта.

      Следующий график зависимости скорости от времени представляет движение тела, движущегося с постоянной скоростью и, следовательно, с нулевым ускорением.

      График зависимости скорости от времени для тела, движущегося с постоянной скоростью, Нилабхро Датта, Study Smarter Originals

      Как видим, значение составляющей скорости остается постоянным и не меняется со временем.

      На следующем графике показано движение тела, движущегося с постоянным (ненулевым) ускорением.

      График зависимости скорости от времени для тела, движущегося с постоянным ускорением, Nilabhro Datta, Study Smart Originals

      Мы можем видеть, как на приведенном выше графике скорость увеличивается с постоянной скоростью. Наклон линии дает нам ускорение объекта.

      Уравнения с постоянным ускорением

      Для тела, движущегося в одном направлении с постоянным ускорением, существует набор из пяти часто используемых уравнений, которые используются для решения пяти различных переменных. Переменные:

      1) s = перемещение

      2) u = начальная скорость

      3) v = конечная скорость

      4) a = ускорение

      5) t = затраченное время

      Уравнения известны как уравнения постоянного ускорения или уравнения SUVAT.

      Уравнения SUVAT

      Существует пять различных уравнений SUVAT, которые используются для соединения и решения указанных выше переменных в системе с постоянным ускорением по прямой.

      1) v = u + at

      2) s = ½ (u + v) t

      3) s = ut + ½at²

      4) s = vt – ½at²

      5) v² = u² + 2 as

      Обратите внимание, что каждое уравнение имеет четыре из пяти переменных SUVAT. Таким образом, учитывая любую из трех переменных, можно было бы найти решение для любой из двух других переменных.

      Автомобиль начинает разгоняться со скоростью 4 м/с² и через 5 секунд врезается в стену со скоростью 40 м/с. На каком расстоянии находилась стена, когда автомобиль начал ускоряться?

      Решение

      Здесь v = 40 м/с, t = 5 секунд, a = 4 м/с². 150 м в течение 5 секунд. Какое расстояние он прошел, прежде чем остановился?

      Решение

      Здесь u = 15 м/с, v = 0 м/с, t = 5 секунд.

      s = ½ (u + v) t

      Решение для s:

      s = ½ (15 + 0) 5 = 37,5 м

      Постоянное ускорение свободного падения

      Сила тяжести Земли вызывает все объекты, чтобы ускориться к нему. Как мы уже говорили, объект, падающий с высоты, падает практически с постоянным ускорением. Если мы пренебрежем эффектами сопротивления воздуха и почти ничтожным гравитационным притяжением других объектов, это будет совершенно постоянное ускорение. Ускорение свободного падения также не зависит от массы объекта.

      Константа g используется для представления ускорения свободного падения. Она примерно равна 9,8 м/с². Если вы решаете задачи, требующие использования значения ускорения свободного падения, вам следует использовать значение g = 9,8 м/с², если только вам не предоставлено более точное измерение.

      Тело, падающее с высоты, можно рассматривать как тело, ускоряющееся с ускорением g. Тело, подброшенное вверх с начальной скоростью, можно рассматривать как тело, замедляющееся со скоростью g, пока оно не достигнет максимальной высоты, на которой ускорение равно нулю. Когда объект падает после достижения максимальной высоты, он снова ускоряется со скоростью g при опускании.

      Кошка, сидящая на стене высотой 2,45 метра, видит мышь на полу и спрыгивает, пытаясь ее поймать. Через сколько времени кошка приземлится на пол?

      Решение

      Здесь u = 0 м/с, s = 2,45 м, a = 9,8 м/с².

      s = ut + ½at²

      Подставляя все значения для решения для t:

      => 2,45 = 0 × t + ½ × 9,8 × t²

      => 2,45 = 4,9t²

      => t = t = 0,71 секунды

      Мяч подброшен вверх с начальной скоростью 26 м/с. Через какое время мяч достигнет максимальной высоты? Предположим, что g = 10 м/с².

      Решение

      Здесь u = 26 м/с, v = 0 м/с, a = -10 м/с².

      V = U + в

      Заменить все значения в уравнение:

      => 0 = 26 – 10t

      Решение для T

      => T = 2,6 секунды

      есть изменение скорости во времени. Если скорость изменения скорости тела остается постоянной во времени, то говорят о постоянном ускорении.

    • Движение объекта можно изобразить графически. Двумя обычно используемыми типами графиков для этой цели являются графики перемещения-времени и графики скорости-времени.

    • Существует пять общих уравнений движения, используемых в системе, включающей постоянное прямолинейное ускорение. Они широко известны как уравнения SUVAT.

    • Тело, падающее с высоты, можно рассматривать как тело, ускоряющееся со скоростью g (константа ускорения свободного падения). Тело, подброшенное вверх с начальной скоростью, можно рассматривать как тело, замедляющееся со скоростью g, пока не достигнет максимальной высоты.

    Часто задаваемые вопросы о постоянном ускорении

    Ускорение свободного падения является постоянным для всех объектов, находящихся вблизи поверхности Земли, поскольку оно зависит от массы Земли, которая является постоянной величиной.

    Ускорение — это изменение скорости во времени. Если скорость изменения скорости тела остается постоянной во времени, то говорят о постоянном ускорении.

    Вы можете рассчитать постоянное ускорение, разделив изменение скорости на затраченное время. Следовательно, a = (v – u)/t, где a = ускорение, v = конечная скорость, u = начальная скорость и t = затраченное время.

    Скорость — это перемещение в единицу времени, тогда как ускорение — это изменение этой скорости в единицу времени.

    Существует пять часто используемых уравнений движения с постоянным ускорением vt – ½at²

    5)    v² = u² + 2 как

    , где s= перемещение, u= начальная скорость, v= конечная скорость, a= ускорение, t= затраченное время.

    Заключительный тест на постоянное ускорение

    Вопрос

    Каковы пять переменных SUVAT?

    Показать ответ

    Ответ

     s= Перемещение, u= Начальная скорость, v= Конечная скорость, a= Ускорение, t= Затраченное время

    Показать вопрос

    Вопрос

    скорость, v = конечная скорость, и попросили найти ускорение. Какое уравнение следует использовать?

    Показать ответ

    Ответ

    v² = u² + 2 как

    Показать вопрос

    Вопрос

    Вам предоставлено s = перемещение, u = начальная скорость, a = ускорение, и вас попросили найти время. Какое уравнение следует использовать?

    Показать ответ

    Ответ

    с = ut + ½at²

    Показать вопрос

    Вопрос

    Вам предоставлены u= начальная скорость, v= конечная скорость, t= время, затраченное и заданное для определения перемещения. Какое уравнение следует использовать?

    Показать ответ

    Ответ

    s = ½ (u + v) t

    Показать вопрос

    Вопрос

    Вам предоставлено u= начальная скорость, a= ускорение, t= затраченное время, и вас просят найти окончательную скорость. Какое уравнение следует использовать?

    Показать ответ

    Ответ

    v = u + at

    Показать вопрос

    Вопрос

    Вам дано s = перемещение, v = конечная скорость, t = затраченное время, и вас попросили найти ускорение. Какое уравнение следует использовать?

    Показать ответ

    Ответ

    s = vt – ½at²

    Показать вопрос

    Вопрос

    Если значение g равно 10 м/с², через сколько времени мяч (находящийся в состоянии покоя) выпадет из большая высота для достижения скорости 50 м/с.

    Показать ответ

    Ответ

    v = u + at

    => 50 = 0 + 10t

    => t = 5 секунд врезается в стену со скоростью 50 м/с через 5 секунд. На каком расстоянии находилась стена, когда автомобиль начал ускоряться?

    Показать ответ

    Ответ

    s = vt – ½at²

    =>  s = 50 × 5 – ½ × 4 × 5² = 200

    Показать вопрос

    Вопрос

    Автомобиль движется по круговой трассе постоянное ускорение и известная начальная скорость. Можете ли вы использовать уравнения SUVAT, чтобы узнать перемещение через заданное время?

    Показать ответ

    Ответить

    Нет, уравнения СУВАТ применимы только к телам, движущимся по прямой, а не по окружности.

    Показать вопрос

    Вопрос

    Футболист выполняет высокоинтенсивную тренировку, бегая на полной скорости в течение определенного периода времени, затем замедляясь в течение определенного периода времени, а затем снова двигаясь на полной скорости в течение определенного периода времени. время. Можете ли вы использовать уравнения SUVAT для описания его движения на протяжении всего курса его обучения?

    Показать ответ

    Ответить

    Нет, потому что ускорение непостоянно.

    Показать вопрос

    Вопрос

    Водитель нажимает на тормоз, и автомобиль останавливается со скорости 15 м/с за 5 секунд. Какое расстояние он прошел, прежде чем остановился?

    Показать ответ

    Ответ

    s = ½ (u + v) t

    => s = ½ (15 + 0) 5 = 37,5 м

    Показать вопрос

    Вопрос

    9000 Велосипедист едет прямо Дорога. Она разгоняется с постоянной скоростью от скорости 4 м/с до скорости 7 м/с² за 30 секунд. Найдите расстояние, которое она пройдет за эти 30 секунд.

    Показать ответ

    Ответ

    s = ½ (u + v) t

    => s = ½ × (4 + 7) × 30 12 м/с замедляется с постоянной скоростью 1,5 м/с², пока не остановится. Как долго движется частица?

    Показать ответ

    Ответ

    v = u +at

    => 0 = 12 – 1,5t

    => t = 8 секунд

    Показать вопрос

    Вопрос

    Частица, движущаяся со скоростью 12 м/с, замедляется с постоянной скоростью 1,5 м/с², пока не остановится. Какое расстояние пролетит частица?

    Показать ответ

    Ответ

    V² = U² + 2 AS

    => 0² = 12² – 2 × 1,5 × S

    => 3S = 144

    => S = 48 M

    Покажите вопрос

    Вопрос.

    Мяч брошен вверх с начальной скоростью 40 м/с. За какое время мяч достигнет максимальной высоты, если g = 10 м/с²?

    Показать ответ

    Ответ

    v = u + at

    => 0 = 40 – 10t

    => t = 4 секунды

    Показать вопрос

    Вопрос

    9000 Что такое постоянное ускорение?

    Показать ответ

    Ответить

    Ускорение определяется как изменение скорости во времени. Если скорость изменения скорости тела остается постоянной во времени, то говорят о постоянном ускорении.

    Показать вопрос

    Вопрос

    Приведите пример постоянного ускорения.

    Показать ответ

    Ответ

    Тело, свободно падающее под действием силы тяжести, на которое не действует никакая другая внешняя сила график времени?

    Показать ответ

    Ответ

    Вычислить наклон линии

    Показать вопрос

    Вопрос

    На графике смещение-время показана прямая линия, параллельная оси времени. Каково ускорение?

    Показать ответ

    Ответ

    Показать вопрос

    Вопрос

    График смещение-время показывает прямую линию, параллельную оси времени. Какова средняя скорость?

    Показать ответ

    Ответ

    Показать вопрос

    Вопрос

    График зависимости скорости от времени показывает прямую линию, параллельную оси времени. Каково ускорение?

    Показать ответ

    Ответ

    Показать вопрос

    Вопрос

    График зависимости скорости от времени представляет собой прямую линию. Каково ускорение?

    Показать ответ

    Ответ

    Ускорение равно градиенту линии

    Показать вопрос

    Вопрос

    График зависимости скорости от времени изогнут и часто меняет направление. Как найти ускорение в данный момент времени?

    Показать ответ

    Ответ

    Рассчитать градиент графика в этой точке

    Показать вопрос

    Вопрос

    Какие пять различных переменных SUVAT?

    Показать ответ

    Ответ

    s= Перемещение, u= Начальная скорость, v= Конечная скорость, a= Ускорение, t= Затраченное время ускорения и известной начальной скорости. Можете ли вы использовать уравнения SUVAT, чтобы узнать его перемещение через заданное время?

    Показать ответ

    Ответить

    Нет, уравнения СУВАТ применимы только к телам, движущимся по прямой, а не по окружности.

    Показать вопрос

    Вопрос

    Вам предоставлено u = начальная скорость, v = конечная скорость, t = время, затраченное на поиск смещения. Какое уравнение следует использовать?

    Показать ответ

    Ответ

    s = ½ (u + v) t

    Показать вопрос

    Вопрос

    Какова величина ускорения свободного падения?

    Показать ответ

    Ответ

    Приблизительно 9,8 м/с²

    Показать вопрос

    Вопрос

    Через какое время мяч (находящийся в состоянии покоя), брошенный с большой высоты, достигнет скорости 29,4 м/с ?

    Показать ответ

    Ответ

    v = u + at

    => 29,4 = 0 + 9,8t

    => t = 3 секунды 20 м/с. Через какое время мяч достигнет максимальной высоты? Предположим, что g=10 м/с².

    Показать ответ

    Ответ

    v = u + at

    => 0 = 20 – 10t

    => t = 2 секунды

    Показать вопрос

    Вопрос

    Мяч брошен с начальной скоростью вверх 20 м/с. Какой высоты пика он достигнет? Предположим, что g=10 м/с².

    Показать ответ

    Ответ

     v² = u² + 2 as

    => 0 = 20² – 2 × 10 × s

    => s = 20 м

    Показать вопрос

    3

    Кот и слон одновременно прыгают с крыши здания. Кто упадет на землю раньше? (Игнорируйте влияние сопротивления воздуха.)

    Показать ответ

    Ответ

    Оба ударятся о землю одновременно, так как ускорение свободного падения не зависит от массы падающего объекта.

    Показать вопрос

    Вопрос

    Какой тип графика используется для отображения движения?

    Показать ответ

    Ответить

    Двумя наиболее часто используемыми графиками для отображения движения являются графики перемещения-времени и графики скорости-времени.

    Показать вопрос

    Вопрос

    Объект движется прямолинейно с постоянным ускорением 5 м/с². Какие из следующих кривых отображают это на графике смещения во времени?

    Показать ответ

    Ответ

    Горизонтальная прямая

    Показать вопрос

    Вопрос

    Объект движется прямолинейно с постоянным ускорением 5 м/с². Какие из следующих кривых отражают это на графике скорость-время?

    Показать ответ

    Ответ

    Горизонтальная прямая

    Показать вопрос

    Вопрос

    Объект движется по прямой с постоянной скоростью. Какие из следующих кривых отображают это на графике смещения во времени?

    Показать ответ

    Ответ

    Горизонтальная прямая

    Показать вопрос

    Вопрос

    Объект движется по прямой с постоянной скоростью. Какие из следующих кривых отражают это на графике скорость-время?

    Показать ответ

    Ответ

    Горизонтальная прямая

    Показать вопрос

    Вопрос

    Объект неподвижен. Какие из следующих кривых отражают это на графике скорость-время?

    Показать ответ

    Ответ

    Горизонтальная прямая

    Показать вопрос

    Вопрос

    Как рассчитать общее пройденное расстояние по заданному графику скорость-время?

    Показать ответ

    Ответ

    Вычислить площадь, заключенную между графиком и осью времени

    Показать вопрос

    Вопрос

    Какую величину мы можем получить, вычислив градиент на графике смещения-времени?

    Показать ответ

    Ответ

    ускорение

    Показать вопрос

    Вопрос

    Какую величину мы можем получить, вычислив градиент на графике скорость-время?

    Показать ответ

    Ответ

    Ускорение

    Показать вопрос

    Вопрос

    Показано, что автомобиль разгоняется от 0 м/с до 25 м/с за 80 секунд по прямой на графике зависимости скорости от времени. Какое расстояние проехал автомобиль за это время?

    Показать ответ

    Ответить

    Пройденное расстояние = площадь, заключенная на графике = основание × высота / 2 = 80 × 25 /2 = 1000 м.

    Показать вопрос

    Вопрос

    Автомобиль движется с постоянной скоростью 13 м/с в течение 40 секунд на графике зависимости скорости от времени. Какое расстояние проехал автомобиль за это время?

    Показать ответ

    Ответить

    Пройденное расстояние = площадь, заключенная на графике = 40 × 13 = 520 м.

    Показать вопрос

    Вопрос

    Бегун бежит по круговой беговой дорожке со скоростью 7 м/с. Какой из следующих графиков будет линейным?

    Показать ответ

    Ответ

    График смещения-времени

    Показать вопрос

    Вопрос

    Мужчина садится в свою машину, но машина отказывается заводиться. Каким будет результирующий график зависимости скорости от времени, изображающий движение автомобиля?

    Показать ответ

    Ответить

    Прямая линия, совпадающая с осью времени (скорость = 0)

    Показать вопрос

    Вопрос

    Человек садится в свою машину, но машина отказывается заводиться. Каким будет результирующий график смещения во времени, изображающий движение автомобиля?

    Показать ответ

    Ответ

    Прямая, совпадающая с осью времени (смещение = 0)

    Показать вопрос

    Вопрос

    График зависимости скорости от времени показывает прямую линию, параллельную оси времени. Каково ускорение?

    Показать ответ

    Ответ

    Показать вопрос

    Вопрос

    График смещение-время показывает прямую линию, параллельную оси времени. Каково ускорение?

    Показать ответ

    Ответ

    Показать вопрос

    Вопрос

    График смещение-время показывает прямую линию, параллельную оси времени. Какова средняя скорость?

    Показать ответ

    Ответ

    Показать вопрос

    Подробнее о постоянном ускорении

    Откройте для себя подходящий контент для ваших предметов

    Не нужно обманывать, если у вас есть все необходимое для успеха! Упаковано в одно приложение!

    Учебный план

    Будьте идеально подготовлены вовремя с индивидуальным планом.

    Тесты

    Проверьте свои знания с помощью игровых тестов.

    Карточки

    Создавайте и находите карточки в рекордно короткие сроки.

    Заметки

    Создавайте красивые заметки быстрее, чем когда-либо прежде.

    Учебные наборы

    Все учебные материалы в одном месте.

    Документы

    Загружайте неограниченное количество документов и сохраняйте их в Интернете.

    Study Analytics

    Определите сильные и слабые стороны вашего исследования.

    Еженедельные цели

    Ставьте индивидуальные учебные цели и зарабатывайте баллы за их достижение.

    Умные напоминания

    Хватит откладывать на потом наши напоминания об учебе.

    Награды

    Зарабатывайте очки, открывайте значки и повышайте уровень во время учебы.

    Волшебный маркер

    Создавайте карточки в заметках полностью автоматически.

    Интеллектуальное форматирование

    Создавайте самые красивые учебные материалы, используя наши шаблоны.

    Узнайте об отрицательном ускорении | Chegg.com

    Отрицательное ускорение Определение

    Отрицательное ускорение, также известное как замедление, представляет собой явление, при котором скорость объекта уменьшается с течением времени с постоянной скоростью. Проще говоря, замедление определяется как противоположность ускорению.

    Обзор отрицательного ускорения

    Замедление также известно как отрицательное ускорение. Он определяется как скорость, с которой движущийся объект замедляется. Замедление можно получить, взяв разницу начальной скорости и конечной скорости объекта. Результат будет включен с отрицательным знаком, так как скорость уменьшается. Формулу ускорения можно использовать для расчета отрицательного ускорения, но результат должен иметь отрицательный знак.

    Для заданной начальной скорости, конечной скорости и времени замедление равно

    rm{Начальная\скорость}}}}{{{\rm{Время}}}}Замедление=ВремяКонечная скорость-Начальная скорость​

    ad=vf-vita_d = \frac{{{v_f} – {v_i}}}{ t}ad​=tvf​−vi​​

    Где ada_d ad​ – замедление, vfv_fvf​ – конечная скорость, viv_ivi​ – начальная скорость, а t – время.

    Есть вопрос по этой теме?

    What you’ll learn:

    • Negative Acceleration Definition
    • Overview of Negative Acceleration
    • The Concept of Deceleration
    • Deceleration as a Vector
    • Examples of Deceleration

    The Concept of Deceleration

    While travelling in a транспортного средства, если ощущается поступательное движение относительно движения транспортного средства, это означает, что тело замедляется. Торможение — это частный случай ускорения, при котором скорость изменения скорости продолжает уменьшаться.

    Ускорение является векторной величиной, что означает, что ускорение имеет не только направление, но и величину. Для одномерного движения направление тела указывается двумя знаками, положительным и отрицательным. Проще говоря, если ускорение имеет отрицательный знак, то объект имеет отрицательное ускорение или замедляется. Отрицательное ускорение вычисляется путем деления конечной скорости за вычетом начальной скорости на время, необходимое телу для уменьшения скорости. Если скорость имеет положительное направление, отрицательное ускорение указывает на то, что объект замедляется. Однако, если скорость имеет отрицательное направление, то отрицательное ускорение указывает на то, что объект ускоряется. Отрицательное ускорение также называется ускорением в отрицательном направлении.

    Замедление как вектор

    Поскольку ускорение является векторной величиной, направление ускорения совпадает с изменением скорости. Поскольку скорость также является вектором, можно изменить величину или направление скорости, то есть ускорение — это изменение либо скорости, либо направления, либо того и другого. С другой стороны, для медленно движущегося объекта направление ускорения будет противоположно его движению. Это известно как замедление, подобно тому, как замедляется поезд, приближающийся к станции.

    Замедление всегда определяется как ускорение в направлении, противоположном направлению скорости. Замедление всегда приводит к снижению скорости. Отрицательное ускорение называется ускорением в отрицательном направлении в выбранной системе координат.

    На графике зависимости скорости от времени линия с положительным наклоном представляет собой ускорение, тогда как линия с отрицательным наклоном представляет собой отрицательное ускорение.

    Рассмотрим приведенный выше график зависимости скорости от времени. Там мы видим, что скорость частицы увеличивается с течением времени от 0 до 2 секунд. Таким образом, мы заключаем, что частица имеет положительное ускорение и наклон кривой положительный в этом интервале времени. В интервале времени от 2 до 3 секунд скорость частицы постоянна, а наклон кривой равен 0. В этот интервал времени ускорение частицы равно нулю.

    Оставить комментарий